++
++
++
++
The advantages of endoscopy over barium radiography in the evaluation of dysphagia include all of the following EXCEPT:
++
++
++
A. Ability to assess function and morphology
++
++
B. Ability to intervene as well as diagnose
++
++
C. Ability to obtain biopsy specimens
++
++
D. Increased sensitivity for the detection of abnormalities identified by color, e.g. Barrett metaplasia
++
++
E. Increased sensitivity for the detection of mucosal lesions
+
++
The answer is A. (Chap. 347) Endoscopy, also known as esophagogastroduodenoscopy (EGD) is the best test for evaluation of the proximal gastrointestinal tract. Because of high-quality images, disorders of color such as Barrett metaplasia and mucosal irregularities are easily demonstrated. Sensitivity of endoscopy is superior to that of barium radiography for mucosal lesions. Because the endoscope has an instrumentation channel, biopsy specimens are easily obtained, and dilation of strictures can also be performed. The sensitivity of radiography compared with endoscopy for detecting reflux esophagitis reportedly ranges from 22%–95%, with higher grades of esophagitis (i.e., ulceration or stricture) exhibiting greater detection rates. Conversely, the sensitivity of barium radiography for detecting esophageal strictures is greater than that of endoscopy, especially when the study is done in conjunction with barium-soaked bread or a 13-mm barium tablet. Barium studies also provide an assessment of esophageal function and morphology that may be undetected on endoscopy. The major shortcoming of barium radiography is that it rarely obviates the need for endoscopy. Barium radiography does not require sedation, which in some populations at risk for conscious sedation is an important consideration.
++
++
++
A 47-year-old man is evaluated in the emergency department for chest pain that developed at a restaurant after swallowing a piece of steak. He reports intermittent episodes of meat getting stuck in his lower chest over the past 3 years, but none as severe as this event. He denies food regurgitation outside of these episodes or heartburn symptoms. He is able to swallow liquids without difficulty and has not had any weight loss. Which of the following is the most likely diagnosis?
++
++
++
++
++
B. Adenocarcinoma of the esophagus
++
++
C. Esophageal diverticula
++
++
D. Plummer-Vinson syndrome
++
++
+
++
The answer is E. (Chap. 347) Intermittent solid food dysphagia is a classic symptom in Schatzki ring in which a distal esophageal ring occurs at the squamocolumnar mucosal junction. The origin of these rings is unknown, and smaller rings with a lumen of greater than 13 mm are common in the general population (up to 15%). When the lumen is less than 13 mm, dysphagia may occur. Schatzki rings typically occur in persons older than 40 years and often cause “steakhouse syndrome” from meat getting stuck at the ring. The rings are easily treated with dilation. Plummer-Vinson syndrome also includes esophageal rings, but typically, the rings occur in the proximal esophagus, are associated with iron-deficiency anemia, and occur in middle-aged women. Achalasia involves both solid and liquid dysphagia often with regurgitation. Adenocarcinoma often includes solid and liquid dysphagia at later stages. Most esophageal diverticulae are asymptomatic.
++
++
++
Which of the following has a well-established association with gastroesophageal reflux?
++
++
++
++
++
++
++
++
++
D. Recurrent aspiration pneumonia
++
++
+
++
The answer is B. (Chap. 347) Aside from the discomfort and local complications of gastroesophageal reflux disease (GERD), a number of other, non–gastrointestinal (GI)-related sites may have complications related to GERD. Syndromes with well-established association with GERD include chronic cough, laryngitis, asthma, and dental erosions. Other diseases have implicated GERD as potentially contributory, but the role of GERD is less well established. These include pharyngitis, pulmonary fibrosis, chronic sinusitis, cardiac arrhythmias, sleep apnea, and recurrent aspiration pneumonia.
++
++
++
A 36-year-old woman with acquired immunodeficiency syndrome (AIDS) and a CD4 count of 35/μL presents with odynophagia and progressive dysphagia. The patient reports daily fevers and a 20-lb weight loss. The patient has been treated with clotrimazole troches without relief. On physical examination, the patient is cachectic with a body mass index (BMI) of 16 and a weight of 86 lb. The patient has a temperature of 38.2°C (100.8°F). She is noted to be orthostatic by blood pressure and pulse. Examination of the oropharynx reveals no evidence of thrush. The patient undergoes esophagogastroduodenoscopy (EGD), which reveals serpiginous ulcers in the distal esophagus without vesicles. No yellow plaques are noted. Multiple biopsies are taken that show intranuclear and intracytoplasmic inclusions in large endothelial cells and fibroblasts. What is the best treatment for this patient’s esophagitis?
++
++
++
++
++
++
++
++
++
++
++
+
++
The answer is A. (Chap. 347) This patient has symptoms of esophagitis. In patients with human immunodeficiency virus (HIV), various infections can cause this disease, including herpes simplex virus (HSV), cytomegalovirus (CMV), varicella-zoster virus (VZV), Candida, and HIV itself. The lack of thrush does not rule out Candida as a cause of esophagitis, and EGD is necessary for diagnosis. CMV classically causes serpiginous ulcers in the distal esophagus that may coalesce to form large ulcers. Brushings alone are insufficient for diagnosis, and biopsies must be performed. Biopsies reveal intranuclear and intracytoplasmic inclusions with enlarged nuclei in large fibroblasts and endothelial cells. Given this patient’s notable swallowing symptoms, intravenous ganciclovir is the treatment of choice. Valganciclovir is an effective oral preparation. Foscarnet is useful in treating ganciclovir-resistant CMV. HSV manifests as vesicles and punched-out lesions in the esophagus with the characteristic finding on biopsy of ballooning degeneration with ground-glass changes in the nuclei. It can be treated with acyclovir or foscarnet in resistant cases. Candida esophagitis has the appearance of yellow nodular plaques with surrounding erythema. Treatment usually requires fluconazole therapy. Finally, HIV alone can cause esophagitis that can be quite resistant to therapy. On EGD, these ulcers appear deep and linear. Treatment with thalidomide or oral glucocorticoids is employed, and highly active antiretroviral therapy should be considered.
++
++
++
A 43-year-old man presents with 6 months of worsening dysphagia and postprandial regurgitation. He reports difficulty and pain with swallowing both liquids and solids, he has no difficulty with the initial components of swallowing but reports pain in the mid-chest region. He will frequently regurgitate undigested food 20–60 minutes after eating or drinking. In the past 2 months, he has lost 15 lb. He also has had one episode of presumed pneumonia 4 months ago notable for a right lower lobe infiltrate. He has no significant past medical history, takes no medications, and does not smoke cigarettes. He works as a service representative at a major electronics store and has never left the United States. Other than signs of recent weight loss, his physical examination is unremarkable. A barium swallow is performed and shown in Figure VIII-5 below. Which of the following is the most likely cause of his disease?
+
++
++
+
++
++
++
A. Autoimmune reaction to latent herpes virus
++
++
B. Diffuse spasm on smooth muscle
++
++
C. Infection by Trypanosoma cruzi
++
++
D. Malignant growth of columnar epithelial cells
++
++
E. Malignant growth squamous epithelial cells
+
++
The answer is A. (Chap. 347) The barium swallow image demonstrates achalasia with esophageal dilation narrowing at the gastroesophageal junction and an air-fluid level in the mid-esophagus. Achalasia is a rare disease caused by loss of ganglion cells within the esophageal myenteric plexus with a population incidence of about 1:100,000; it usually presents between age 25 and 60. With long-standing disease, aganglionosis is noted. The disease involves both excitatory (cholinergic) and inhibitory (nitric oxide) ganglionic neurons. This leads to impaired deglutitive lower esophageal sphincter (LES) relaxation and absent peristalsis. Increasing evidence suggests that the ultimate cause of ganglion cell degeneration in achalasia is an autoimmune process attributable to a latent infection with human HSV-1 combined with genetic susceptibility. Long-standing achalasia is characterized by progressive dilatation and sigmoid deformity of the esophagus with hypertrophy of the LES. Clinical manifestations may include dysphagia, regurgitation, chest pain, and weight loss. Most patients report solid and liquid food dysphagia. Regurgitation occurs when food, fluid, and secretions are retained in the dilated esophagus. Patients with advanced achalasia are at risk for bronchitis, pneumonia, or lung abscess from chronic regurgitation and aspiration. The differential diagnosis of achalasia includes diffuse esophageal spasm, Chagas disease, and pseudoachalasia. Chagas disease is endemic in areas of central Brazil, Venezuela, and northern Argentina and spread by the bite of the reduviid (kissing) bug that transmits the protozoan Trypanosoma cruzi. The chronic phase of the disease develops years after infection and results from destruction of autonomic ganglion cells throughout the body, including the heart, gut, urinary tract, and respiratory tract. Tumor infiltration, most commonly seen with carcinoma in the gastric fundus or distal esophagus, can mimic idiopathic achalasia. The resultant “pseudoachalasia” accounts for up to 5% of suspected cases and is more likely with advanced age, abrupt onset of symptoms (<1 year), and weight loss. Hence, endoscopy is a necessary part of the evaluation of achalasia. When the clinical suspicion for pseudoachalasia is high and endoscopy nondiagnostic, computed tomography (CT) scanning or endoscopic ultrasound may be of value. There is no known way of preventing or reversing achalasia. Therapy is directed at reducing LES pressure so that gravity and esophageal pressurization promote esophageal emptying. Peristalsis rarely, if ever, recovers. Botulinum toxin, injected into the LES under endoscopic guidance, inhibits acetylcholine release from nerve endings and improves dysphagia in about 66% of cases for at least 6 months. The only durable therapies for achalasia are pneumatic dilatation and Heller myotomy. Pneumatic dilatation, with a reported efficacy ranging from 32%–98%, is an endoscopic technique using a noncompliant, cylindrical balloon dilator positioned across the LES and inflated to a diameter of 3–4 cm. The major complication is perforation, with a reported incidence of 0.5%–5%.
++
++
++
A 43-year-old man presents with 6 months of worsening dysphagia and postprandial regurgitation. He reports difficulty and pain with swallowing both liquids and solids, he has no difficulty with the initial components of swallowing but reports pain in the mid-chest region. He will frequently regurgitate undigested food 20–60 minutes after eating or drinking. In the past 2 months, he has lost 15 lb. He also has had one episode of presumed pneumonia 4 months ago notable for a right lower lobe infiltrate. He has no significant past medical history, takes no medications, and does not smoke cigarettes. He works as a service representative at a major electronics store and has never left the United States. Other than signs of recent weight loss, his physical examination is unremarkable. A barium swallow is performed and shown in Figure VIII-5 below. In the patient described, which of the following is the most efficacious therapy?
+
++
++
+
++
++
++
++
++
B. Calcium channel blocker
++
++
++
++
++
++
+
++
The answer is A. (Chap. 347) The barium swallow image demonstrates achalasia with esophageal dilation narrowing at the gastroesophageal junction and an air-fluid level in the mid-esophagus. Achalasia is a rare disease caused by loss of ganglion cells within the esophageal myenteric plexus with a population incidence of about 1:100,000; it usually presents between age 25 and 60. With long-standing disease, aganglionosis is noted. The disease involves both excitatory (cholinergic) and inhibitory (nitric oxide) ganglionic neurons. This leads to impaired deglutitive lower esophageal sphincter (LES) relaxation and absent peristalsis. Increasing evidence suggests that the ultimate cause of ganglion cell degeneration in achalasia is an autoimmune process attributable to a latent infection with human HSV-1 combined with genetic susceptibility. Long-standing achalasia is characterized by progressive dilatation and sigmoid deformity of the esophagus with hypertrophy of the LES. Clinical manifestations may include dysphagia, regurgitation, chest pain, and weight loss. Most patients report solid and liquid food dysphagia. Regurgitation occurs when food, fluid, and secretions are retained in the dilated esophagus. Patients with advanced achalasia are at risk for bronchitis, pneumonia, or lung abscess from chronic regurgitation and aspiration. The differential diagnosis of achalasia includes diffuse esophageal spasm, Chagas disease, and pseudoachalasia. Chagas disease is endemic in areas of central Brazil, Venezuela, and northern Argentina and spread by the bite of the reduviid (kissing) bug that transmits the protozoan Trypanosoma cruzi. The chronic phase of the disease develops years after infection and results from destruction of autonomic ganglion cells throughout the body, including the heart, gut, urinary tract, and respiratory tract. Tumor infiltration, most commonly seen with carcinoma in the gastric fundus or distal esophagus, can mimic idiopathic achalasia. The resultant “pseudoachalasia” accounts for up to 5% of suspected cases and is more likely with advanced age, abrupt onset of symptoms (<1 year), and weight loss. Hence, endoscopy is a necessary part of the evaluation of achalasia. When the clinical suspicion for pseudoachalasia is high and endoscopy nondiagnostic, computed tomography (CT) scanning or endoscopic ultrasound may be of value. There is no known way of preventing or reversing achalasia. Therapy is directed at reducing LES pressure so that gravity and esophageal pressurization promote esophageal emptying. Peristalsis rarely, if ever, recovers. Botulinum toxin, injected into the LES under endoscopic guidance, inhibits acetylcholine release from nerve endings and improves dysphagia in about 66% of cases for at least 6 months. The only durable therapies for achalasia are pneumatic dilatation and Heller myotomy. Pneumatic dilatation, with a reported efficacy ranging from 32%–98%, is an endoscopic technique using a noncompliant, cylindrical balloon dilator positioned across the LES and inflated to a diameter of 3–4 cm. The major complication is perforation, with a reported incidence of 0.5%–5%.
++
++
++
A 64-year-old man with a long history of abdominal pain, heartburn, and dyspepsia has an EGD to evaluate for peptic ulcer disease. No gastric or duodenal ulcers are found, but there are tongues of reddish mucosa extending proximally from the gastroesophageal junction into the esophagus. Biopsies of these areas demonstrate columnar metaplasia. All of the following statements regarding this diagnosis are true EXCEPT:
++
++
++
A. Finding high-grade dysplasia mandates further intervention.
++
++
B. High-dose proton pump inhibitor therapy will likely cause regression of the mucosal abnormalities.
++
++
C. The incidence of these lesions has increased in the era of potent acid suppression.
++
++
D. The patient has a high risk of coexisting cancer.
++
++
E. The patient is at significant risk of esophageal adenocarcinoma.
+
++
The answer is B. (Chap. 347) Barrett metaplasia is the most serious complication of GERD. It has a strong association with the subsequent development of esophageal adenocarcinoma. The incidence of these lesions has increased, not decreased, in the era of potent acid suppression. Barrett metaplasia is endoscopically recognized by tongues of reddish mucosa extending proximally from the gastroesophageal junction or histopathologically identified by the finding of specialized columnar metaplasia. Barrett metaplasia can progress to adenocarcinoma through the intermediate stages of low- and high-grade dysplasia. Due to this risk, areas of Barrett metaplasia and especially any included areas of mucosal irregularity should be extensively biopsied. No high-level evidence confirms that aggressive antisecretory therapy or antireflux surgery causes regression of Barrett esophagus or prevents adenocarcinoma. Although the management of Barrett esophagus remains controversial, the finding of dysplasia in Barrett esophagus, particularly high-grade dysplasia, mandates further intervention. In addition to the high rate of progression to adenocarcinoma, there is also a high prevalence of unrecognized coexisting cancer with high-grade dysplasia. Nonetheless, treatment remains controversial. Esophagectomy, intensive endoscopic surveillance, and mucosal ablation have all been advocated. Currently, esophagectomy is the gold standard treatment for high-grade dysplasia in an otherwise healthy patient with minimal surgical risk. However, esophagectomy has a mortality ranging from 3%–10%, along with substantial morbidity. As a result of these factors and the increasing evidence of the effectiveness of endoscopic therapy with purpose-built radiofrequency ablation devices, many now favor this therapy as a preferable management strategy.
++
++
++
A 57-year-old man is evaluated with an EGD after an episode of hematemesis. The patient reports a history of tobacco use and hypercholesterolemia but is otherwise healthy. He has had lower back pain for the past month and has been intermittently using acetaminophen 1000 mg for relief. His endoscopy shows a 3-cm duodenal ulcer. Which of the following statements is correct regarding this finding?
++
++
++
A. The lesion should be biopsied because duodenal ulcers have an elevated risk of being due to carcinoma.
++
++
B. First-line therapy should be discontinuation of acetaminophen use.
++
++
C. The patient is not at risk for any associated cancers.
++
++
D. Poor socioeconomic status is a risk factor for development of this condition.
++
++
E. Antral gastritis is rarely found with this condition.
+
++
The answer is D. (Chap. 348) The patient has a duodenal ulcer, which is almost universally due to Helicobacter pylori infection, although in a minority of cases, nonsteroidal anti-inflammatory drug (NSAID) use may either facilitate development or be the only identified cause. The patient was taking acetaminophen and not a traditional NSAID, making H pylori–associated peptic ulcer disease the most likely cause of the findings. H pylori infection is closely correlated with advancing age, low socioeconomic status, and low education levels. After initial infection, antral gastritis is common, and in a portion of patients, duodenal or gastric ulcers form. Associated with these conditions is the development of gastric cancer or mucosa-associated lymphoid tissue (MALT) lymphoma. Duodenal ulcers are rarely cancerous, whereas this is a not an uncommon finding in gastric ulcers. After discovery of the ulcer, first-line therapy is eradication of H pylori in addition to acid suppression.
++
++
++
A 58-year-old man is evaluated for abdominal pain by his primary care physician. He reports severe stress at the job for the last 3 months and has since noted that he has epigastric pain that is relieved by eating and drinking milk. He has not had food regurgitation, dysphagia, or bloody emesis or bowel movements. He denies any symptoms in his chest. Peptic ulcer disease is suspected. Which of the following statements regarding noninvasive testing for Helicobacter pylori is true?
++
++
++
A. There is no reliable noninvasive method to detect H pylori.
++
++
B. Stool antigen testing is not appropriate for either diagnosis of or proof of cure after therapy for H pylori.
++
++
C. Plasma antibodies to H pylori offer the greatest sensitivity for diagnosis of infection.
++
++
D. Expose to low-dose radiation is a limitation to urea breath test.
++
++
E. False-negative test results using the urea breath test may occur with recent use of nonsteroidal anti-inflammatory drugs (NSAIDs).
+
++
The answer is D. (Chap. 348) Noninvasive testing for H pylori infection is recommended in patients with suggestive symptoms and no other indication for endoscopy (e.g., GI bleeding, atypical symptoms). Several tests have good sensitivity and specificity, including plasma serology for H pylori, 14C- or 13C-urea breath test, and the fecal H pylori antigen test (Table VIII-9). Sensitivity and specificity are >80% and >90%, respectively, for serology, whereas the sensitivity and specificity of the urea breath test and fecal antigen testing are >90% for both. Serology is not useful for early follow-up after therapy completion because antibody titers will take several weeks to months to fall. The urea breath test, which relies on the presence of urease secreted by H pylori to digest the swallowed radioactive urea and liberate 14C or 13C as part of ammonia, is simple and rapid. It is useful for early follow-up because it requires living bacteria to secrete urease and produce a positive test. The limitations to the test include requirement for ingestion of radioactive materials, albeit low dose, and false-negative results with recent use of proton pump inhibitors, antibiotics, or bismuth compounds.
++
++
++
++
A 44-year-old woman complains of 6 months of epigastric pain that is worst between meals. She also reports symptoms of heartburn. The pain is typically relieved by over-the-counter antacid medications. She comes to clinic after noting her stools darkening. She has no significant past medical history and takes no medications. Her physical examination is normal except for diffuse mid-epigastric pain. Her stools are heme positive. She undergoes EGD, which demonstrates a well-circumscribed, 2-cm duodenal ulcer that is positive for H pylori. Which of the following is the recommended initial therapy given these findings?
++
++
++
++
++
++
++
++
++
++
++
+
++
The answer is A. (Chap. 348) Documented eradication of H pylori in patients with peptic ulcer disease (PUD) is associated with a dramatic decrease in ulcer recurrence to <10%–20% as compared to 59% in gastric ulcer patients and 67% in duodenal ulcer patients when the organism is not eliminated. Eradication of the organism may lead to diminished recurrent ulcer bleeding. The effect of its eradication on ulcer perforation is unclear. Extensive effort has been made in determining who of the many individuals with H pylori infection should be treated. The common conclusion arrived at by multiple consensus conferences around the world is that H pylori should be eradicated in patients with documented PUD. This holds true independent of time of presentation (first episode or not), severity of symptoms, presence of confounding factors such as ingestion of NSAIDs, or whether the ulcer is in remission. Multiple drugs have been evaluated in the therapy of H pylori. No single agent is effective in eradicating the organism. Combination therapy for 14 days provides the greatest efficacy, although regimens based on sequential administration of antibiotics also appear promising. A shorter administration course (7–10 days), although attractive, has not proved as successful as the 14-day regimens. Suggested treatment regimens for H pylori are outlined in Table VIII-10. Choice of a particular regimen will be influenced by several factors, including efficacy, patient tolerance, existing antibiotic resistance, and cost of the drugs. The aim for initial eradication rates should be 85%–90%. Dual therapy (proton pump inhibitor [PPI] plus amoxicillin, PPI plus clarithromycin, ranitidine bismuth citrate [Tritec] plus clarithromycin) is not recommended in view of studies demonstrating eradication rates of <80%–85%. Addition of acid suppression assists in providing early symptom relief and enhances bacterial eradication. Triple therapy, although effective, has several drawbacks, including the potential for poor patient compliance and drug-induced side effects. Compliance is being addressed by simplifying the regimens so that patients can take the medications twice a day. Simpler (dual therapy) and shorter regimens (7 and 10 days) are not as effective as triple therapy for 14 days. Two anti–H pylori regimens are available in prepackaged formulation: Prevpac (lansoprazole, clarithromycin, and amoxicillin) and Helidac (bismuth subsalicylate, tetracycline, and metronidazole). The contents of the Prevpac are to be taken twice per day for 14 days, whereas Helidac constituents are taken four times per day with an antisecretory agent (PPI or H2 blocker), also taken for at least 14 days. Clarithromycin-based triple therapy should be avoided in settings where H pylori resistance to this agent exceeds 15%–20%. Quadruple therapy should be reserved for patients with failure to eradicate H pylori after an effective initial course.
++
++
++
++
A 57-year-old man with peptic ulcer disease experiences transient improvement with H pylori eradication. However, 3 months later, symptoms recur despite acid-suppressing therapy. He does not take NSAIDs. Stool analysis for H pylori antigen is negative. Upper gastrointestinal (GI) endoscopy reveals prominent gastric folds together with the persistent ulceration in the duodenal bulb previously detected and the beginning of a new ulceration 4 cm proximal to the initial ulcer. Fasting gastrin levels are elevated, and basal acid secretion is 15 mEq/hr. What is the best test to perform to make the diagnosis?
++
++
++
A. No additional testing is necessary.
++
++
B. Blood sampling for gastrin levels following a meal
++
++
C. Blood sampling for gastrin levels following secretin administration
++
++
D. Endoscopic ultrasonography of the pancreas
++
++
E. Genetic testing for mutations in the MEN1 gene
+
++
The answer is C. (Chap. 348) Fasting gastrin levels can be elevated in a variety of conditions, including atrophic gastritis with or without pernicious anemia, G-cell hyperplasia, and acid suppressive therapy (gastrin levels increase as a consequence of loss of negative feedback). The diagnostic concern in a patient with persistent ulcers following optimal therapy is Zollinger-Ellison syndrome (ZES). The result is not sufficient to make a diagnosis because gastrin levels may be elevated in a variety of conditions. Elevated basal acid secretion also is consistent with ZES, but up to 12% of patients with PUD may have basal acid secretion as high as 15 mEq/hr. Thus, additional testing is necessary. Gastrin levels may go up with a meal (>200%), but this test does not distinguish G-cell hyperfunction from ZES. The best test in this setting is the secretin stimulation test. An increase in gastrin levels >200 pg within 15 minutes of administering 2 µg/kg of secretin by intravenous bolus has a sensitivity and specificity of >90% for ZES. Endoscopic ultrasonography is useful in locating the gastrin-secreting tumor once the positive secretin test is obtained. Genetic testing for mutations in the gene that encodes the menin protein can detect the fraction of patients with gastrinomas that are a manifestation of multiple endocrine neoplasia type 1 (Wermer syndrome). Gastrinoma is the second most common tumor in this syndrome after parathyroid adenoma, but its peak incidence is generally in the third decade.
++
++
++
A 65-year-old man presented to the hospital 2 weeks ago with an acute abdomen, hypotension, anemia, and respiratory failure. His past medical history was notable for hypertension and hypercholesterolemia for which he took enalapril and atorvastatin. At laparotomy, he was found to have a perforated duodenal ulcer with peritonitis and hemoperitoneum. A vagotomy and Billroth I anastomosis were performed. He has improved gradually and is increasing his oral intake and ambulation. His white blood cell (WBC) count and hemoglobin are normal. This afternoon approximately 3 hours after lunch, he reported the acute onset of lightheadedness, confusion, palpitations, and diaphoresis. His temperature is 36.0°C, heart rate is 110 bpm, blood pressure is 120/70 mmHg, and oxygen saturation is 95% on room air. Which of the following is most likely present?
++
++
++
++
++
++
++
C. Pulmonary embolism on helical chest computed tomography (CT)
++
++
D. Seizure focus on electroencephalogram (EEG)
++
++
E. ST elevation on electrocardiogram (ECG)
+
++
The answer is B. (Chap. 348) Surgical intervention in PUD can be viewed as being either elective, for treatment of medically refractory disease, or as urgent/emergent, for the treatment of an ulcer-related complication. The development of pharmacologic and endoscopic approaches for the treatment of PUD and its complications has led to a substantial decrease in the number of operations needed for this disorder, with a decrease of over 90% for elective ulcer surgery over the last four decades. Refractory ulcers are an exceedingly rare occurrence. Surgery is more often required for treatment of an ulcer-related complication. Free peritoneal perforation occurs in ~2%–3% of DU patients. As in the case of bleeding, up to 10% of these patients will not have antecedent ulcer symptoms. Concomitant bleeding may occur in up to 10% of patients with perforation, with mortality being increased substantially. The procedure that provides the lowest rates of ulcer recurrence (1%) but has the highest complication rate is vagotomy (truncal or selective) in combination with antrectomy. Antrectomy is aimed at eliminating an additional stimulant of gastric acid secretion, gastrin. Two principal types of reanastomoses are used after antrectomy: gastroduodenostomy (Billroth I) and gastrojejunostomy (Billroth II). Dumping syndrome consists of a series of vasomotor and GI signs and symptoms and occurs in patients who have undergone vagotomy and drainage (especially Billroth procedures). Two phases of dumping, early and late, can occur. Early dumping takes place 15–30 minutes after meals and consists of crampy abdominal discomfort, nausea, diarrhea, belching, tachycardia, palpitations, diaphoresis, light-headedness, and, rarely, syncope. These signs and symptoms arise from the rapid emptying of hyperosmolar gastric contents into the small intestine, resulting in a fluid shift into the gut lumen with plasma volume contraction and acute intestinal distention. Release of vasoactive GI hormones (vasoactive intestinal polypeptide, neurotensin, motilin) is also theorized to play a role in early dumping. The late phase of dumping typically occurs 90 minutes to 3 hours after meals. Vasomotor symptoms (light-headedness, diaphoresis, palpitations, tachycardia, and syncope) predominate during this phase. This component of dumping is thought to be secondary to hypoglycemia from excessive insulin release. Dumping syndrome is most noticeable after meals rich in simple carbohydrates (especially sucrose) and high osmolarity. Ingestion of large amounts of fluids may also contribute. Up to 50% of postvagotomy and drainage patients will experience dumping syndrome to some degree early on. Signs and symptoms often improve with time, but a severe protracted picture can occur in up to 1% of patients. Although this patient is certainly at risk of pulmonary embolism and myocardial infarction, his symptoms are typical of hypoglycemia due to dumping syndrome.
++
++
++
A 65-year-old man presented to the hospital 2 weeks ago with an acute abdomen, hypotension, anemia, and respiratory failure. His past medical history was notable for hypertension and hypercholesterolemia for which he took enalapril and atorvastatin. At laparotomy, he was found to have a perforated duodenal ulcer with peritonitis and hemoperitoneum. A vagotomy and Billroth I anastomosis were performed. He has improved gradually and is increasing his oral intake and ambulation. His white blood cell (WBC) count and hemoglobin are normal. This afternoon approximately 3 hours after lunch, he reported the acute onset of lightheadedness, confusion, palpitations, and diaphoresis. His temperature is 36.0°C, heart rate is 110 bpm, blood pressure is 120/70 mmHg, and oxygen saturation is 95% on room air. In the patient described, which of the following is the most likely diagnosis?
++
++
++
++
++
++
++
++
++
D. Inferior wall myocardial infarction
++
++
+
++
The answer is C. (Chap. 348) Surgical intervention in PUD can be viewed as being either elective, for treatment of medically refractory disease, or as urgent/emergent, for the treatment of an ulcer-related complication. The development of pharmacologic and endoscopic approaches for the treatment of PUD and its complications has led to a substantial decrease in the number of operations needed for this disorder, with a decrease of over 90% for elective ulcer surgery over the last four decades. Refractory ulcers are an exceedingly rare occurrence. Surgery is more often required for treatment of an ulcer-related complication. Free peritoneal perforation occurs in ~2%–3% of DU patients. As in the case of bleeding, up to 10% of these patients will not have antecedent ulcer symptoms. Concomitant bleeding may occur in up to 10% of patients with perforation, with mortality being increased substantially. The procedure that provides the lowest rates of ulcer recurrence (1%) but has the highest complication rate is vagotomy (truncal or selective) in combination with antrectomy. Antrectomy is aimed at eliminating an additional stimulant of gastric acid secretion, gastrin. Two principal types of reanastomoses are used after antrectomy: gastroduodenostomy (Billroth I) and gastrojejunostomy (Billroth II). Dumping syndrome consists of a series of vasomotor and GI signs and symptoms and occurs in patients who have undergone vagotomy and drainage (especially Billroth procedures). Two phases of dumping, early and late, can occur. Early dumping takes place 15–30 minutes after meals and consists of crampy abdominal discomfort, nausea, diarrhea, belching, tachycardia, palpitations, diaphoresis, light-headedness, and, rarely, syncope. These signs and symptoms arise from the rapid emptying of hyperosmolar gastric contents into the small intestine, resulting in a fluid shift into the gut lumen with plasma volume contraction and acute intestinal distention. Release of vasoactive GI hormones (vasoactive intestinal polypeptide, neurotensin, motilin) is also theorized to play a role in early dumping. The late phase of dumping typically occurs 90 minutes to 3 hours after meals. Vasomotor symptoms (light-headedness, diaphoresis, palpitations, tachycardia, and syncope) predominate during this phase. This component of dumping is thought to be secondary to hypoglycemia from excessive insulin release. Dumping syndrome is most noticeable after meals rich in simple carbohydrates (especially sucrose) and high osmolarity. Ingestion of large amounts of fluids may also contribute. Up to 50% of postvagotomy and drainage patients will experience dumping syndrome to some degree early on. Signs and symptoms often improve with time, but a severe protracted picture can occur in up to 1% of patients. Although this patient is certainly at risk of pulmonary embolism and myocardial infarction, his symptoms are typical of hypoglycemia due to dumping syndrome.
++
++
++
A 23-year-old woman is evaluated by her primary care physician for diffuse, crampy abdominal pain. She reports that she has had abdominal pain for the last several years, but it is getting worse and now associated with intermittent diarrhea without flatulence. This does not waken her at night. Stools do not float and are not hard to flush. She has not noted any worsening with specific foods, but she does have occasional rashes on her lower legs. She has lost about 10 lb over the last year. She is otherwise healthy and takes no medications. Which of the following is the most appropriate recommendation at this point?
++
++
++
A. Increased dietary fiber intake
++
++
B. Measurement of antiendomysial antibody
++
++
C. Measurement of 24-hour fecal fat
++
++
D. Referral to gastroenterologist for endoscopy
++
++
E. Trial of lactose-free diet
+
++
The answer is B. (Chap. 349) The patient presents with nonspecific GI symptoms, but the presence of weight loss does suggest malabsorption syndrome. Patients with lactose intolerance are usually able to relate symptoms to consumption of milk-based products and also report a strong history of crampy pain and flatulence. Therefore, a lactose-free diet is unlikely to be helpful. The patient does not have nocturnal diarrhea, which is commonly a feature of steatorrhea along with floating stools. In the absence of symptoms suggesting fat malabsorption, the first test should not be fecal fat measurement. Because the patient has weight loss, irritable bowel syndrome is less likely, and increased dietary fiber is unlikely to be useful. Finally, the patient’s symptoms may be consistent with celiac disease. The widespread availability of antibodies to gliadin, endomysium, and tissue transglutaminase can be easily measured in peripheral blood. Antiendomysial antibody has a 90%–95% sensitivity and equal specificity, making it a reasonable first test in symptomatic individuals. The presence of the antibody is not diagnostic, however, and duodenal biopsy is recommended. Duodenal biopsy will show villous atrophy, absence or reduced height of villi, cuboidal appearance of surface epithelial cells, and increased lymphocytes and plasma cells in the lamina propria. These changes regress with complete removal of gluten from the diet.
++
++
++
All of the following are direct complications of short bowel syndrome EXCEPT:
++
++
++
A. Cholesterol gallstones
++
++
B. Coronary artery disease
++
++
C. Gastric acid hypersecretion
++
++
D. Renal calcium oxalate calculi
++
++
+
++
The answer is B. (Chap. 349) Short bowel syndrome is a descriptive term referring to the many clinical complications that may occur after resection of varying lengths of the small bowel. Rarely, these complications may be due to congenital abnormalities of the small bowel. Most commonly, in adults, short bowel syndrome occurs in mesenteric vascular disease, primary mucosal or submucosal disease (Crohn disease), and operations without preexisting small bowel disease such as trauma. Multiple factors contribute to diarrhea and steatorrhea including gastric acid hypersecretion, increased bile acids in the colon due to absent or decreased reabsorption in the small bowel, and lactose intolerance due to increased gastric acid secretion. Nonintestinal symptoms may include renal calcium oxalate calculi due to an increase in oxalate absorption by the large intestine with subsequent hyperoxaluria. This may be due to increased fatty acids in the colon that bind calcium, and therefore, calcium in the gut is not free to bind oxalate and free oxalate is thus absorbed in the large intestine. Increased bile acid pool size results in the generation of cholesterol gallstones from supersaturation in gallbladder bile. Gastric hypersecretion of acid is well described and thought to be due to loss of inhibition of gastric acid secretion because of absent short bowel, which secretes inhibitory hormones. Coronary artery disease is not described as a complication of short bowel syndrome.
++
++
++
A 54-year-old man is evaluated by a gastroenterologist for diarrhea that has been present for approximately 1 month. He reports stools that float and are difficult to flush down the toilet; these can occur at any time of day or night but seem worsened by fatty meals. In addition, he reports pain in many joints lasting days to weeks and not relieved by ibuprofen. His wife notes that the patient has had difficulty with memory for the last few months. He has lost 30 pounds and reports intermittent low-grade fevers. He takes no medications and is otherwise healthy. Endoscopy is recommended. Which of the following is the most likely finding on small bowel biopsy?
++
++
++
++
++
B. Flat villi with crypt hyperplasia
++
++
C. Mononuclear cell infiltrate in the lamina propria
++
++
D. Normal small bowel biopsy
++
++
E. Periodic acid–Schiff (PAS)–positive macrophages containing small bacilli
+
++
The answer is E. (Chap. 349) The patient presents with symptoms suggestive of Whipple disease, a chronic multisystem disease often including diarrhea/steatorrhea, migratory arthralgias, weight loss, and central nervous system (CNS) or cardiac problems. Generally the presentation is of insidious onset, and dementia is a late finding and poor prognostic sign. The disease primarily occurs in middle-aged white males. The diagnosis requires small bowel biopsy and demonstration of periodic acid–Schiff (PAS)–positive macrophages within the small bowel. Small bacilli are often present and suggest the diagnosis of Whipple disease. Similar macrophages may be found in other affected organs (e.g., the CNS). Dilated lymphatics are present in patients with intestinal lymphangiectasia. Mononuclear cell infiltrate in the lamina propria is often demonstrated in patients with tropical sprue, and flat villi with crypt hyperplasia is the hallmark of celiac disease.
++
++
++
A 54-year-old man presents with 1 month of diarrhea. He states that he has 8–10 loose bowel movements a day. He has lost 8 lb during this time. Vital signs and physical examination are normal. Serum laboratory studies are normal. A 24-hour stool collection reveals 500 g of stool with a measured stool osmolality of 200 mOsmol/L and a calculated stool osmolality of 210 mOsmol/L. Based on these findings, what is the most likely cause of this patient’s diarrhea?
++
++
++
++
++
++
++
++
++
D. Vasoactive intestinal peptide tumor
++
++
+
++
The answer is D. (Chap. 349) This patient has a stool osmolality gap (measured stool osmolality – calculated stool osmolality) of 10 mOsmol/L, suggesting a secretory rather than an osmotic cause for diarrhea. Secretory causes of diarrhea include toxin-mediated diarrhea (cholera, enterotoxigenic Escherichia coli) and intestinal peptide–mediated diarrhea in which the major pathophysiology is a luminal or circulating secretagogue. The distinction between secretory diarrhea and osmotic diarrhea aids in forming a differential diagnosis. Secretory diarrhea will not decrease substantially during a fast and has a low osmolality gap. Osmotic diarrhea will generally decrease during a fast and has a high (>50 mOsmol/L) osmolality gap. Celiac sprue, chronic pancreatitis, lactase deficiency, and Whipple disease all cause an osmotic diarrhea. A low stool osmolality (<290 mOsmol/kg H2O) reflects the addition of either dilute urine or water, indicating either collection of urine and stool together or so-called factitious diarrhea, a form of Münchausen syndrome.
++
++
++
Which of the following GI disorders is characterized by increased absorption from the GI tract into the portal circulation?
++
++
++
++
++
++
++
++
++
++
++
+
++
The answer is C. (Chap. 349) Almost all GI malabsorption clinical problems are associated with diminished intestinal absorption of one or more dietary nutrients and are often referred to as the malabsorption syndrome. Most malabsorption syndromes are associated with steatorrhea, an increase in stool fat excretion to >6% of dietary fat intake. The only clinical conditions in which absorption is increased are hemochromatosis and Wilson disease, in which absorption of iron and copper, respectively, is elevated. Celiac disease may cause significant malabsorption of multiple nutrients, with diarrhea, steatorrhea, weight loss, and the consequences of nutrient depletion (i.e., anemia and metabolic bone disease) or depletion of a single nutrient (e.g., iron or folate deficiency, osteomalacia, edema from protein loss). Malabsorption of bile salts and vitamins is common in Crohn disease due to ileal involvement. The magnitude of malabsorption is dependent on the extent of disease. Whipple disease is a chronic multisystemic disease associated with diarrhea, steatorrhea, weight loss, arthralgia, and CNS and cardiac problems; it is caused by the bacteriumTropheryma whipplei.
++
++
++
Which of the following statements regarding the epidemiology of inflammatory bowel disease is correct?
++
++
++
A. Monozygotic twins are highly concordant for ulcerative colitis.
++
++
B. Oral contraceptive use decreases the incidence of Crohn disease.
++
++
C. Persons of Asian descent have the highest rates of ulcerative colitis and Crohn disease.
++
++
D. Smoking may decrease the incidence of ulcerative colitis.
++
++
E. Typical age of onset for Crohn disease is 40–50 years old.
+
++
The answer is D. (Chap. 351) The incidence of inflammatory bowel disease is highly influenced by ethnicity, location, and environmental factors. Both conditions have their highest incidence in the United Kingdom and North America, and the peak incidence has a bimodal distribution of age of presentation: 15–30 years and 60–80 years of age. Incidence of both ulcerative colitis and Crohn disease is highest among persons of Ashkenazi Jewish population. Prevalence decreases progressively in non-Jewish white, African American, Hispanic, and Asian populations. Cigarette smoking is associated with a decreased incidence of ulcerative colitis but may cause Crohn disease. Oral contraceptive use is associated with a slightly higher incidence of Crohn disease but not ulcerative colitis. Monozygotic twins are highly concordant for Crohn disease but not ulcerative colitis.
++
++
++
A 24-year-old woman is admitted to the hospital with a 1-year history of severe abdominal pain and chronic diarrhea that has been bloody for the past 2 months. She reports a 20-lb weight loss, frequent fevers, and night sweats. She denies vomiting. Her abdominal pain is crampy and primary involves her right lower quadrant. She is otherwise healthy. Examination is concerning for an acute abdomen with rebound and guarding present. CT shows free air in the peritoneum. She is urgently taken to the operating room for surgical exploration, where she is found to have multiple strictures and a perforation of her bowel in the terminal ileum. The rectum was spared, and a fissure from the duodenum to the jejunum is found. The perforated area is resected and adhesions lysed. Which of the following findings on pathology of her resected area confirms her diagnosis?
++
++
++
++
++
++
++
C. Noncaseating granuloma throughout the bowel wall
++
++
D. Special stain for Clostridium difficile toxin
++
++
E. Transmural acute and chronic inflammation
+
++
The answer is C. (Chap. 351) Chronic, bloody diarrhea associated with weight loss and systemic symptoms in a young person is highly suggestive of inflammatory bowel disease. The patient’s surgical findings suggest discontinuous lesions, which is typical of Crohn disease. Ulcerative colitis, in contrast, typically affects the rectum and proceeds caudally from there without normal mucosa until the area of inflammation terminates. The presence of strictures and fissures further supports the diagnosis of Crohn disease, as these are not features of ulcerative colitis. Microscopically, both ulcerative colitis and Crohn disease may have crypt abscess, and although Crohn disease is more often transmural, full-thickness disease may be present in ulcerative colitis. The hallmark of Crohn disease is granulomas that may be present throughout the bowel wall and involve the lymph nodes, mesentery, peritoneum, liver, and pancreas. Although pathognomonic for Crohn disease, granulomas are only found in about half of surgical resections. Flat villi are not always present in either disease and are more commonly found in isolation with celiac disease.
++
++
++
A 45-year-old man with ulcerative colitis has been treated for the past 5 years with infliximab with excellent resolution of his bowel symptoms and endoscopic evidence of normal colonic mucosa. He is otherwise healthy. He is evaluated by a dermatologist for a lesion that initially was a pustule over his right lower extremity but has since progressed in size with ulceration. The ulcer is moderately painful. He does not recall any trauma to the area. On examination, the ulcer measures 15 × 7 cm, and central necrosis is present. The edges of the ulcer are violaceous. No other lesions are identified. Which of the following is the most likely diagnosis?
++
++
++
++
++
B. Metastatic Crohn disease
++
++
++
++
++
++
+
++
The answer is D. (Chap. 351) There are a number of dermatologic manifestations of inflammatory bowel disease (IBD), and each type of IBD has a particular predilection for different dermatologic conditions. This patient has pyoderma gangrenosum. Pyoderma gangrenosum can occur in up to 12% of patients with ulcerative colitis and is characterized by a lesion that begins as a pustule and progresses concentrically to surrounding normal skin. The lesions ulcerate with violaceous, heaped margins and surrounding erythema. They are typically found on the lower extremities. Often the lesions are difficult to treat and respond poorly to colectomy; similarly, pyoderma gangrenosum is not prevented by colectomy. Treatment commonly includes intravenous antibiotics, glucocorticoids, dapsone, infliximab, and other immunomodulatory agents. Erythema nodosum is more common in Crohn disease and attacks correlate with bowel symptoms. The lesions are typically multiple, red hot, tender nodules measuring 1–5 cm and are found on the lower legs and arms. Psoriasis is more common in ulcerative colitis. Finally, pyoderma vegetans is a rare disorder in intertriginous areas reported to be a manifestation of IBD in the skin.
++
++
++
Inflammatory bowel disease (IBD) may be caused by exogenous factors. GI flora may promote an inflammatory response or may inhibit inflammation. Probiotics have been used to treat IBD. Which of the following organisms has been used in the treatment of IBD?
++
++
++
++
++
++
++
++
++
++
++
+
++
The answer is D. (Chap. 351 and Cochrane Database Syst Rev 2007 Oct 17; [4]) Despite being described as a clinical entity for over a century, the etiology of IBD remains cryptic. Current theory is related to an interplay between inflammatory stimuli in genetically predisposed individuals. Recent studies have identified a group of genes or polymorphisms that confer risk of IBD. Multiple microbiologic agents, including some that reside as “normal” flora, may initiate IBD by triggering an inflammatory response. Anaerobic organisms (e.g., Bacteroides and Clostridia spp.) may be responsible for the induction of inflammation. Other organisms, for unclear reasons, may have the opposite effect. These “probiotic” organisms include Lactobacillus spp., Bifidobacterium spp., Taenia suis, and Saccharomyces boulardii. Shigella, Escherichia, and Campylobacter spp. are known to promote inflammation. Studies of probiotic therapy in adults and children with IBD have shown potential benefit for reducing disease activity.
++
++
++
Your 33-year-old patient with Crohn disease has had a disappointing disease response to glucocorticoids and 5-aminosalicylic acid (5-ASA) agents. He is interested in steroid-sparing agents. He has no liver or renal disease. You prescribe once-weekly methotrexate injections. In addition to monitoring hepatic function and complete blood count, what other complication of methotrexate therapy do you advise the patient of?
++
++
++
A. Disseminated histoplasmosis
++
++
++
++
++
++
++
++
E. Primary sclerosing cholangitis
+
++
The answer is D. (Chap. 295) Methotrexate, azathioprine, cyclosporine, tacrolimus, and anti–tumor necrosis factor (TNF) antibody are reasonable options for patients with Crohn disease, depending on the extent of macroscopic disease. Pneumonitis is a rare but serious complication of methotrexate therapy. Primary sclerosing cholangitis is an extraintestinal manifestation of IBD. Pancreatitis is an uncommon complication of azathioprine, and IBD patients treated with azathioprine are at fourfold increased risk of developing a lymphoma. Anti-TNF antibody therapy is associated with an increased risk of tuberculosis, disseminated histoplasmosis, and a number of other infections.
++
++
++
All of the following statements regarding the risk of cancer in patients with IBD are correct EXCEPT:
++
++
++
A. Patients with Crohn disease are at greater risk for hematologic malignancies than the general population.
++
++
B. Patients with Crohn disease are at lower risk of GI malignancies than patients with ulcerative colitis.
++
++
C. Patients with long-standing ulcerative colitis are at increased risk of developing carcinoma of the colon.
++
++
D. Screening colonoscopy is recommended every 1–2 years in patients with a >8- to 10-year history of extensive ulcerative colitis regardless of age.
++
++
E. Ulcerative colitis patients with high-grade dysplasia found on colonoscopy should undergo immediate colectomy.
+
++
The answer is B. (Chap. 351) Patients with long-standing ulcerative colitis (UC) are at increased risk for developing colonic epithelial dysplasia and carcinoma. The risk of neoplasia in chronic UC increases with duration and extent of disease. From one large meta-analysis, the risk of cancer in patients with UC is estimated at 2% after 10 years, 8% after 20 years, and 18% after 30 years of disease. Data from a 30-year surveillance program in the United Kingdom calculated the risk of colorectal cancer to be 7.7% at 20 years and 15.8% at 30 years of disease. The rates of colon cancer are higher than in the general population, and colonoscopic surveillance is the standard of care. Annual or biennial colonoscopy with multiple biopsies is recommended for patients with >8–10 years of extensive colitis (greater than one-third of the colon involved) or 12–15 years of proctosigmoiditis (less than one-third but more than just the rectum). The cancer risks in Crohn disease and UC are probably equivalent for similar extent and duration of disease. Thus, the same endoscopic surveillance strategy used for UC is recommended for patients with chronic Crohn colitis. If flat high-grade dysplasia is encountered on colonoscopic surveillance, the usual treatment is colectomy for UC and either colectomy or segmental resection for Crohn disease. If flat low-grade dysplasia is found, most investigators recommend immediate colectomy. Patients with Crohn disease may have an increased risk of non-Hodgkin lymphoma, leukemia, and myelodysplastic syndromes.
++
++
++
Which of the following patients requires no further testing before making the diagnosis of irritable bowel syndrome and initiating treatment?
++
++
++
A. A 76-year-old woman with 6 months of intermittent crampy abdominal pain that is worse with stress and associated with bloating and diarrhea.
++
++
B. A 25-year-old woman with 6 months of abdominal pain, bloating, and diarrhea that has worsened steadily and who now awakes from sleep at night to move her bowels.
++
++
C. A 30-year-old man with 6 months of lower abdominal crampy pain relieved with bowel movements, usually loose. Symptoms are worse during the daytime at work and better on the weekend. Weight loss is not present.
++
++
D. A 19-year-old female college student with 2 months of diarrhea and worsening abdominal pain with occasional blood in her stool.
++
++
E. A 27-year-old woman with 6 months of intermittent abdominal pain, bloating, and diarrhea without associated weight loss. Crampy pain and diarrhea persist after a 48-hour fast.
+
++
The answer is C. (Chap. 352) Irritable bowel syndrome (IBS) is characterized by the following: recurrence of lower abdominal pain with altered bowel habits over a period of time without progressive deterioration, onset of symptoms during periods of stress or emotional upset, absence of other systemic symptoms such as fever and weight loss, and small-volume stool without evidence of blood. Warning signs that the symptoms may be due to something other than IBS include presentation for the first time in old age, progressive course from the time of onset, persistent diarrhea after a 48-hour fast, and presence of nocturnal diarrhea or steatorrheal stools. Each patient, except for patient C, has “warning” symptoms that should prompt further evaluation.
++
++
++
A 29-year-old woman comes to see you in clinic because of abdominal discomfort. She feels abdominal discomfort on most days of the week, and the pain varies in location and intensity. She notes constipation as well as diarrhea, but diarrhea predominates. Compared to 6 months ago, she has more bloating and flatulence than she has had before. She identifies eating and stress as aggravating factors, and her pain is relieved by defecation. You suspect irritable bowel syndrome. Laboratory data include: WBC count 8000/µL, hematocrit 32%, platelets 210,000/µL, and erythrocyte sedimentation rate (ESR) 44 mm/hr. Stool studies show the presence of lactoferrin but no blood. Which intervention is appropriate at this time?
++
++
++
++
++
++
++
++
++
D. Reassurance and patient counseling
++
++
+
++
The answer is C. (Chap. 352) Although this patient has signs and symptoms consistent with IBS, the differential diagnosis is large. Few tests are required for patients who have typical IBS symptoms and no alarm features. In this patient, alarm features include anemia, an elevated erythrocyte sedimentation rate, and evidence of white blood cells in the stool. Alarm features warrant further investigation to rule out other GI disorders such as colonic pathology including diverticular disease or IBD. In this case, colonoscopy to evaluate for luminal lesions and mucosal characteristics would be the logical first step. At this point, with the warning signs, empiric therapy for IBS is premature. Reassurance, stool-bulking agents, and antidepressants are all therapies to consider if a patient does indeed have IBS.
++
++
++
After a careful history and physical and a cost-effective workup, you have diagnosed a 24-year-old woman patient with irritable bowel syndrome. What other condition would you reasonably expect to find in this patient?
++
++
++
A. Abnormal brain anatomy
++
++
++
++
C. History of sexually transmitted diseases
++
++
++
++
E. Sensory hypersensitivity to peripheral stimuli
+
++
The answer is D. (Chap. 352) Up to 80% of patients with IBS also have abnormal psychiatric features; however, no single psychiatric diagnosis predominates. The mechanism is not well understood but may involve altered pain thresholds. Although these patients are hypersensitive to colonic stimuli, this does not carry over to the peripheral nervous system. Functional brain imaging shows disparate activation in, for example, the mid-cingulate cortex, but brain anatomy does not discriminate IBS patients from those without IBS. An association between a history of sexual abuse and IBS has been reported. There is no reported association with sexually transmitted diseases. Patients with IBS do not have an increased risk of autoimmunity.
++
++
++
A 24-year-old woman has had 2 years of abdominal complaints characterized by episodic abdominal pain that is relieved by stooling. She reports that she has frequent small stools often soon after eating. She does not wake at night to use the bathroom. She has treated herself with various over-the-counter medications. She reports that she has tried a variety of diets recommended by daytime television that have occasionally but not consistently helped her symptoms. Her past medical history is notable for mild depression for which she takes fluoxetine. Her physical examination is completely normal, as are her basic metabolic panel, thyroid function, and complete blood count. A diet low in which of the following has been shown to be beneficial in patients such as her?
++
++
++
++
++
++
++
C. Fermentable oligosaccharides, disaccharides, monosaccharides, and polyols
++
++
D. Rice and rice products
++
++
+
++
The answer is C. (Chap. 352) This patient has a diarrhea-predominant form of IBS. Alteration in bowel habits is the most consistent clinical feature in IBS. The most common pattern is constipation alternating with diarrhea, usually with one of these symptoms predominating. At first, constipation may be episodic, but eventually, it becomes continuous and increasingly intractable to treatment with laxatives. Patients whose predominant symptom is constipation may have weeks or months of constipation interrupted with brief periods of diarrhea. In other patients, diarrhea may be the predominant symptom. Diarrhea resulting from IBS usually consists of small volumes of loose stools. Most patients have stool volumes of <200 mL. Nocturnal diarrhea does not occur in IBS. Diarrhea may be aggravated by emotional stress or eating. Stool may be accompanied by passage of large amounts of mucus. Bleeding is not a feature of IBS unless hemorrhoids are present, and malabsorption or weight loss does not occur.
++
Bowel pattern subtypes are highly unstable, with patients frequently alternating between constipation, diarrhea, or a mixed pattern. A diet low in fermentable oligosaccharides, disaccharides, monosaccharides, and polyols (FODMAPs) has been shown to be helpful in IBS patients (Table VIII-28). FODMAPs are poorly absorbed by the small intestine and fermented by bacteria in the colon to produce gas and osmotically active carbohydrates. A randomized controlled study showed that a diet low in FODMAPs reduced symptoms in IBS patients.
++
++
++
++
A 78-year-old woman is admitted to the hospital with fever, loss of appetite, and left lower quadrant pain. She is not constipated but has not moved her bowels recently. Laboratory examination is notable for an elevated WBC count. These symptoms began approximately 3 days ago and have steadily worsened. Which of the following statements regarding the use of radiologic imaging to evaluate her condition is true?
++
++
++
A. Air-fluid levels are commonly seen on plain abdominal films.
++
++
B. Less than 25% of patients present with peritoneal signs.
++
++
C. Lower GI bleeding will likely be visualized on CT angiography.
++
++
D. Thickened colonic wall is not required on CT for the diagnosis of her likely condition.
++
++
E. Ultrasound of the pelvis is the best modality to visualize the likely pathologic process.
+
++
The answer is B. (Chap. 353) The patient presents with classic signs of diverticulitis with fever, abdominal pain that is usually left lower quadrant, anorexia or obstipation, and leukocytosis. This most commonly occurs in older individuals. Patients may present with acute abdomen due to perforation, although this occurs in <25% of cases. Plain radiographs of the abdomen are seldom helpful but may show the presence of an air-fluid level in the left lower quadrant indicating a giant diverticulum with impending perforation. CT with oral contrast is the diagnostic modality of choice with the following findings: sigmoid diverticula, thickened colonic wall >4 mm, and inflammation within the pericolic space with or without the collection of contrast material or fluid. In 16% of patients, an abdominal abscess may be present. Symptoms of IBS may mimic those of diverticulitis. Therefore, suspected diverticulitis that does not meet CT criteria or is not associated with a leukocytosis or fever is not diverticular disease. Other conditions that can mimic diverticular disease include an ovarian cyst, endometriosis, acute appendicitis, and pelvic inflammatory disease. Although the benefit of colonoscopy in the evaluation of patients with diverticular disease has been called into question, its use is still considered important in the exclusion of colorectal cancer. The parallel epidemiology of colorectal cancer and diverticular disease provides enough concern for an endoscopic evaluation before operative management. Therefore, a colonoscopy should be performed ~6 weeks after an attack of diverticular disease. Although diverticular disease may result in hematochezia, these are generally not temporally linked to diverticulitis.
++
++
++
Which of the following patients is MOST appropriate for surgical management of the their acute diverticulitis?
++
++
++
A. A 45-year-old woman with rheumatoid arthritis treated with infliximab and prednisone.
++
++
B. A 63-year-old woman with diverticulitis in the descending colon and a distal stricture.
++
++
C. A 70-year-old woman with end-stage renal disease and colonic wall thickening of 8 mm on CT scan.
++
++
D. A 77-year-old man with two episodes of diverticulitis in the past 2 years.
++
++
E. None of the above patients requires surgical management.
+
++
The answer is B. (Chap. 353) Medical management is appropriate for many patients with uncomplicated diverticular disease. Uncomplicated disease involves fever, abdominal pain, leukocytosis, and anorexia/obstipation, whereas complicated disease is characterized by abscess formation, perforation, strictures, or fistulae. Uncomplicated disease accounts for at least 75% of cases. Medical therapy generally involves bowel rest and antibiotics, usually trimethoprim/sulfamethoxazole or ciprofloxacin and metronidazole targeting aerobic gram-negative rods and anaerobic bacteria. Patients with more than two attacks of diverticulitis were previously thought to require surgical therapy, but newer data suggest that these patients do not have an increased risk of perforation and can continue medical management. Patients with immunosuppressive therapy, chronic renal failure, or collagen vascular disease have a fivefold higher risk of perforation during recurrent attacks. Surgical therapy is indicated for surgical low-risk patients with complicated disease, such as a stricture.
++
++
++
A 67-year-old man is evaluated by the emergency department for blood in the toilet bowel after moving his bowels. Blood was also present on the toilet paper after wiping. He does report straining and recent constipation. He has a history of systemic hypertension and hyperlipidemia. Vital signs are normal, and he is not orthostatic. Anoscopy shows external hemorrhoids, hematocrit is normal, and bleeding does not recur during his 6-hour emergency department stay. Which of the following is the most appropriate management?
++
++
++
++
++
B. Cortisone suppositories and fiber supplementation
++
++
++
++
D. Operative hemorrhoidectomy
++
++
+
++
The answer is B. (Chap. 353) Hemorrhoids can be internal or external; however, they are normally internal and may prolapse to the external position. Hemorrhoids are staged in the following manner: stage I, enlargement with bleeding; stage II, protrusion with spontaneous reduction; stage III, protrusion requiring manual reduction; and stage IV, irreducible protrusion. Stage I, which this patient has, is treated with fiber supplementation, cortisone suppositories, and/or sclerotherapy. Stage II is treated with fiber and cortisone suppositories. Stage III is offered the prior three therapies and banding or operative hemorrhoidectomy. Stage IV patients benefit from fiber and cortisone therapy as well as operative hemorrhoidectomy. Although substantial upper GI bleeding may result in hematochezia, the absence of suggestive signs/symptoms and the consistent findings of hemorrhoids do not indicate the need for upper endoscopy.
++
++
++
Which of the following statements regarding anorectal abscess is true?
++
++
++
A. Anorectal abscess is more common in diabetic patients.
++
++
B. Anorectal abscess is more common in women.
++
++
C. Difficulty voiding is uncommon and should prompt further evaluation of anorectal abscess.
++
++
D. Examination in the operating room under anesthesia is required for adequate exploration in most cases.
++
++
E. The peak incidence is in the seventh decade of life.
+
++
The answer is A. (Chap. 353) An anorectal abscess is an abnormal fluid-containing cavity in the anorectal region. Anorectal abscess results from an infection involving the glands surrounding the anorectal canal. The disease is more common in males, with a peak incidence in the third to fifth decades. Patients with diabetes, with IBD, or who are immunocompromised are at increased risk for this condition. Perianal pain with defecation and fever are common presenting symptoms.
++
++
++
An 88-year-old woman is brought to your clinic by her family because she has become increasingly socially withdrawn. The patient lives alone and has been reluctant to visit or be visited by her family. Family members, including seven children, also note a foul odor in her apartment and on her person. She has not had any weight loss. Alone in the examining room, she only complains of hemorrhoids. On mental status examination, she does have signs of depression. Which of the following interventions is most appropriate at this time?
++
++
++
++
++
B. Treatment with an antidepressant medication
++
++
C. Physical examination including genitourinary and rectal examination
++
++
D. Screening for occult malignancy
++
++
E. Serum thyroid-stimulating hormone
+
++
The answer is C. (Chap. 353) This patient has symptoms (social isolation), signs (foul odor), and risk factors (multiparity) for procidentia (rectal prolapse) and fecal incontinence. Procidentia is far more common in women than men and is often associated with pelvic floor disorders. It is not uncommon for these patients to become socially withdrawn and suffer from depression because of the associated fecal incontinence. The foul odor is a result of poor perianal hygiene due to the prolapsed rectum. Although depression in the elderly is an important medical problem, it is too premature in the evaluation to initiate medical therapy for depression. Occult malignancy and thyroid abnormalities may cause fecal incontinence and depression, but a physical examination would be diagnostic and avoid costly tests. Often patients are concerned they have a rectal mass or carcinoma. Examination after an enema often makes the prolapse apparent. Medical therapy is limited to stool-bulking agents or fiber. Surgical correction is the mainstay of therapy.
++
++
++
An 85-year-old woman is brought to a local emergency department by her family. She has been complaining of abdominal pain off and on for several days, but this morning, she states that this is the worst pain of her life. She is able to describe a sharp, stabbing pain in her abdomen. Her family reports that she has not been eating and seems to have no appetite. She has a past medical history of atrial fibrillation and hypercholesterolemia. She has had two episodes of vomiting and, in the emergency department, experiences diarrhea that is hemoccult positive. On examination, she is afebrile, with a heart rate of 105 bpm and blood pressure of 111/69 mmHg. Her abdomen is mildly distended, and she has hypoactive bowel sounds. She does not exhibit rebound tenderness or guarding. She is admitted for further management. Several hours after admission, she becomes unresponsive. Blood pressure is difficult to obtain and at best approximation is 60/40 mmHg. She has a rigid abdomen. Surgery is called, and the patient is taken for emergent laparotomy. She is found to have acute mesenteric ischemia. Which of the following statements is true regarding this diagnosis?
++
++
++
A. Mortality for this condition is >50%.
++
++
B. Risk factors include low-fiber diet and obesity.
++
++
C. The “gold standard” for diagnosis is CT scan of the abdomen.
++
++
D. The lack of acute abdominal signs in this case is unusual for mesenteric ischemia.
++
++
E. The splanchnic circulation is poorly collateralized.
+
++
The answer is A. (Chap. 354) Mesenteric ischemia is a relatively uncommon and highly morbid illness. Acute mesenteric ischemia is usually due to arterial embolus (usually from the heart) or to thrombosis in a diseased vascular bed. Major risk factors include age, atrial fibrillation, valvular disease, recent arterial catheterization, and recent myocardial infarction. Ischemia occurs when the intestines are inadequately perfused by the splanchnic circulation. This blood supply has extensive collateralization and can receive up to 30% of the cardiac output, making poor perfusion an uncommon event. Patients with acute mesenteric ischemia will frequently present with pain out of proportion to their initial physical examination. As ischemia persists, peritoneal signs and cardiovascular collapse will follow. Mortality is >50%. While radiographic imaging can suggest ischemia, the gold standard for diagnosis is laparotomy.
++
++
++
A 63-year-old man with a history of diabetes and myocardial infarction was admitted to the medical intensive care unit (ICU) 1 day ago with sepsis due to pneumococcal pneumonia with bacteremia. He was started on antibiotics immediately but initially required high doses of noradrenaline and fluids to stabilize his blood pressure. The noradrenaline was weaned off approximately 12 hours ago. Over the past 2 hours, he has had increasing abdominal pain, distension, and bloody stools. His physical examination is notable for blood pressure of 100/50 mmHg, regular heart rate of 100 bpm, respiratory rate of 22 breaths/min, and oxygen saturation of 93% on high-flow nasal oxygen. He has a diffusely tender abdomen with no audible bowel sounds. An abdominal radiograph shows multiple small bowel air-fluid levels. Which of the following is the most likely diagnosis?
++
++
++
++
++
++
++
C. Inflammatory bowel disease
++
++
D. Nonocclusive mesenteric ischemia
++
++
+
++
The answer is D. (Chap. 354) This patient with a history of atherosclerotic cardiovascular disease and diabetes is at high risk for nonocclusive mesenteric ischemia in the setting of sepsis, hypotension, and administered vasoconstrictors. Intestinal ischemia is further classified based on etiology, which dictates management (Table VIII-35): (1) arteria-occlusive mesenteric ischemia, (2) nonocclusive mesenteric ischemia, and (3) mesenteric venous thrombosis. Risk factors for arteria-occlusive mesenteric ischemia are generally acute in onset and include atrial fibrillation, recent myocardial infarction, valvular heart disease, and recent cardiac or vascular catheterization, all of which result in embolic clots reaching the mesenteric circulation. Nonocclusive mesenteric ischemia, also known as “intestinal angina,” is generally more insidious and most often seen in the aging population affected by atherosclerotic disease. Nonocclusive mesenteric ischemia is also seen in patients receiving high-dose vasopressor infusions, patients with cardiogenic or septic shock, and patients with cocaine overdose. Nonocclusive mesenteric ischemia is the most prevalent GI disease complicating cardiovascular surgery. Mesenteric venous thrombosis is less common and is associated with the presence of a hypercoagulable state including protein C or S deficiency, anti–thrombin III deficiency, polycythemia vera, and carcinoma. In the absence of atrial fibrillation, nonocclusive mesenteric ischemia is more likely in this patient. There is no reason to suspect IBD as a cause of the bloody stools in this critically ill patient with known bacteremic sepsis, and the patient has only been on antibiotics for 1 day and is unlikely to have Clostridium difficile colitis at this point.
++
++
++
++
A 74-year-old woman is 2 days status post hip surgery for a fracture after a fall. Her only medication prior to admission was a calcium supplement, and she has no prior surgical history. Over the past 24 hours, she has had increasing abdominal discomfort and distension. She received a dose of cefazolin prior to surgery but no other antibiotics. On physical examination, she is afebrile with blood pressure of 140/80 mmHg, heart rate of 110 bpm, respiratory rate of 16 breaths/min, and oxygen saturation of 100% on 2 L of nasal oxygen. She has a distended tympanic abdomen with absent bowel sounds. There is no rebound tenderness. Her upright abdominal film is shown in Figure VIII-36. Which of the following is the most likely diagnosis?
+
++
++
FIGURE VIII-36 (Reproduced, with permission, from Bongard FS, Sue DY [eds]. Current Critical Care Diagnosis & Treatment. Originally published by Appleton & Lange. Copyright (c) 1994 by The McGraw-Hill Companies, Inc., Fig. 13-18A.)
+
++
++
++
A. Acalculous cholecystitis
++
++
B. Colonic pseudo-obstruction
++
++
C. Perforated duodenal ulcer
++
++
D. Small bowel obstruction
++
++
+
++
The answer is B. (Chap. 355) The radiograph shows massively dilated colon extending to the rectum. This radiograph is consistent with colonic pseudo-obstruction or Ogilvie syndrome. Ogilvie syndrome may be seen in elderly patients after nonabdominal surgery or in patients with underlying autonomic dysfunction. The presence of gas in the colon makes small bowel obstruction unlikely. There is no extraintestinal air that would be suggestive of small or large bowel perforation. Small bowel ileus is characterized by multiple small bowel air-fluid levels on radiograph. The differential for extensive colonic dilation includes toxic megacolon due to C difficile infection. In this case, that is less likely given the recent surgery and lack of antibiotic treatment. Neostigmine is an acetylcholinesterase inhibitor that increases cholinergic (parasympathetic) activity and can stimulate colonic motility. Some studies have shown it to be moderately effective in alleviating acute colonic pseudo-obstruction. It is the most common therapeutic approach and can be used once it is certain that there is no mechanical obstruction. Cardiac monitoring is required, and atropine should be immediately available for symptomatic bradycardia. Intravenous administration induces defecation and flatus within 10 minutes in the majority of patients who will respond. Surgical therapy may be necessary in cases of bowel perforation or impending perforation. Morphine, with its anticholinergic side effects, may worsen small or large bowel pseudo-obstruction. Oral vancomycin is the treatment for C difficile infection.
++
++
++
A 74-year-old woman is 2 days status post hip surgery for a fracture after a fall. Her only medication prior to admission was a calcium supplement, and she has no prior surgical history. Over the past 24 hours, she has had increasing abdominal discomfort and distension. She received a dose of cefazolin prior to surgery but no other antibiotics. On physical examination, she is afebrile with blood pressure of 140/80 mmHg, heart rate of 110 bpm, respiratory rate of 16 breaths/min, and oxygen saturation of 100% on 2 L of nasal oxygen. She has a distended tympanic abdomen with absent bowel sounds. There is no rebound tenderness. Her upright abdominal film is shown in Figure VIII-36. Which of the following is the next recommended therapy for the patient described?
+
++
++
FIGURE VIII-36 (Reproduced, with permission, from Bongard FS, Sue DY [eds]. Current Critical Care Diagnosis & Treatment. Originally published by Appleton & Lange. Copyright (c) 1994 by The McGraw-Hill Companies, Inc., Fig. 13-18A.)
+
++
++
++
++
++
++
++
++
++
++
++
+
++
The answer is D. (Chap. 355) The radiograph shows massively dilated colon extending to the rectum. This radiograph is consistent with colonic pseudo-obstruction or Ogilvie syndrome. Ogilvie syndrome may be seen in elderly patients after nonabdominal surgery or in patients with underlying autonomic dysfunction. The presence of gas in the colon makes small bowel obstruction unlikely. There is no extraintestinal air that would be suggestive of small or large bowel perforation. Small bowel ileus is characterized by multiple small bowel air-fluid levels on radiograph. The differential for extensive colonic dilation includes toxic megacolon due to C difficile infection. In this case, that is less likely given the recent surgery and lack of antibiotic treatment. Neostigmine is an acetylcholinesterase inhibitor that increases cholinergic (parasympathetic) activity and can stimulate colonic motility. Some studies have shown it to be moderately effective in alleviating acute colonic pseudo-obstruction. It is the most common therapeutic approach and can be used once it is certain that there is no mechanical obstruction. Cardiac monitoring is required, and atropine should be immediately available for symptomatic bradycardia. Intravenous administration induces defecation and flatus within 10 minutes in the majority of patients who will respond. Surgical therapy may be necessary in cases of bowel perforation or impending perforation. Morphine, with its anticholinergic side effects, may worsen small or large bowel pseudo-obstruction. Oral vancomycin is the treatment for C difficile infection.
++
++
++
All of the following are potential causes of appendix obstruction and appendicitis EXCEPT:
++
++
++
++
++
++
++
++
++
++
++
+
++
The answer is C. (Chap. 356) Obstruction of the appendiceal lumen is believed to typically result in appendicitis. Although obstruction is most commonly caused by fecalith, which results from accumulation and inspissation of fecal matter around vegetable fibers, other causes have been described. These other potential causes include enlarged lymphoid follicles associated with viral infection (e.g., measles), inspissated barium, worms (e.g., pinworms, Ascaris, and Taenia), and tumors such as carcinoma or carcinoid. Cholelithiasis is a common cause of acute pancreatitis.
++
++
++
A 32-year-old woman is evaluated in the emergency department for abdominal pain. She reports a vague loss of appetite for the past day and has had progressively severe abdominal pain, initially at her umbilicus, but now localized to her right lower quadrant. The pain is crampy. She has not moved her bowels or vomited. She reports that she is otherwise healthy and has had no sick contact. Exam is notable for a temperature of 100.7°F and heart rate of 105 bpm, but otherwise, vital signs are normal. Her abdomen is tender in the right lower quadrant, and pelvic examination is normal. Urine pregnancy test is negative. Which of the following imaging modalities is most likely to confirm her diagnosis?
++
++
++
A. CT of the abdomen without contrast
++
++
++
++
++
++
D. Plain film of the abdomen
++
++
E. Ultrasound of the abdomen
+
++
The answer is A. (Chap. 356) The patient presents with typical findings for acute appendicitis with anorexia, progressing to vague periumbilical pain, followed by localization to the right lower quadrant. Low-grade fever and leukocytosis are frequently present. Although acute appendicitis is primarily a clinical diagnosis, imaging modalities are frequently employed because the symptoms are not always classic. Plain radiographs are rarely helpful except when an opaque fecalith is found in the right lower quadrant (<5% of cases). Ultrasound may demonstrate an enlarged appendix with a thick wall but is most useful to rule out ovarian pathology, tubo-ovarian abscess, or ectopic pregnancy. The effectiveness of ultrasonography as a tool to diagnose appendicitis is highly operator dependent. Even in very skilled hands, the appendix may not be visualized. Its overall sensitivity is 0.86, with a specificity of 0.81. Nonenhanced and contrast-enhanced CT are superior to ultrasound or plain radiograph in the diagnosis of acute appendicitis with sensitivity of 0.94 and specificity of 0.95. Findings often include a thickened appendix with periappendiceal stranding and often the presence of a fecalith (Figure VIII-39). Free air is uncommon, even in the case of a perforated appendix. Nonvisualization of the appendix on CT is associated with surgical findings of a normal appendix 98% of the time. Colonoscopy has no role in the diagnosis of acute appendicitis.
+
++
++
+
++
++
++
A 38-year-old man is seen in the urgent care center with several hours of severe abdominal pain. His symptoms began suddenly, but he reports several months of pain in the epigastrium after eating, with a resultant 10-lb weight loss. He takes no medications besides over-the-counter antacids and has no other medical problems or habits. On physical examination, temperature is 38.0°C (100.4°F), pulse is 130 bpm, respiratory rate is 24 breaths/min, and blood pressure is 110/50 mmHg. His abdomen has absent bowel sounds and is rigid with involuntary guarding diffusely. A plain film of the abdomen is obtained and shows free air under the diaphragm. Which of the following is most likely to be found in the operating room?
++
++
++
++
++
++
++
C. Perforated duodenal ulcer
++
++
D. Perforated gallbladder
++
++
E. Perforated gastric ulcer
+
++
The answer is C. (Chap. 356) The patient presents with several months of epigastric abdominal pain that is worse after eating. His symptoms are highly suggestive of peptic ulcer disease, with the worsening pain after eating suggesting a duodenal ulcer. The current presentation with acute abdomen and free air under the diaphragm points to a diagnosis of perforated viscus. Perforated gallbladder is less likely in light of the duration of symptoms and the absence of the significant systemic symptoms that often accompany this condition. Because the patient is relatively young with no risk factors for mesenteric ischemia, necrotic bowel from an infarction is highly unlikely. Pancreatitis can have a similar presentation, but a pancreas cannot perforate and liberate free air. Peritonitis is most commonly associated with bacterial infection, but it can be caused by the abnormal presence of physiologic fluids, for example, gastric contents, bile, pancreatic enzymes, blood, or urine, or by foreign bodies. In this case, peritonitis is most likely due to the presence of gastric juice in the peritoneal cavity after perforation of a duodenal ulcer has allowed these juices to leave the gut lumen.
++
++
++
A 38-year-old man is seen in the urgent care center with several hours of severe abdominal pain. His symptoms began suddenly, but he reports several months of pain in the epigastrium after eating, with a resultant 10-lb weight loss. He takes no medications besides over-the-counter antacids and has no other medical problems or habits. On physical examination, temperature is 38.0°C (100.4°F), pulse is 130 bpm, respiratory rate is 24 breaths/min, and blood pressure is 110/50 mmHg. His abdomen has absent bowel sounds and is rigid with involuntary guarding diffusely. A plain film of the abdomen is obtained and shows free air under the diaphragm. Which of the following is the mostly likely source of peritonitis in the patient?
++
++
++
++
++
++
++
++
++
++
++
+
++
The answer is D. (Chap. 356) The patient presents with several months of epigastric abdominal pain that is worse after eating. His symptoms are highly suggestive of peptic ulcer disease, with the worsening pain after eating suggesting a duodenal ulcer. The current presentation with acute abdomen and free air under the diaphragm points to a diagnosis of perforated viscus. Perforated gallbladder is less likely in light of the duration of symptoms and the absence of the significant systemic symptoms that often accompany this condition. Because the patient is relatively young with no risk factors for mesenteric ischemia, necrotic bowel from an infarction is highly unlikely. Pancreatitis can have a similar presentation, but a pancreas cannot perforate and liberate free air. Peritonitis is most commonly associated with bacterial infection, but it can be caused by the abnormal presence of physiologic fluids, for example, gastric contents, bile, pancreatic enzymes, blood, or urine, or by foreign bodies. In this case, peritonitis is most likely due to the presence of gastric juice in the peritoneal cavity after perforation of a duodenal ulcer has allowed these juices to leave the gut lumen.
++
++
++
A 61-year-old man is admitted to your service for swelling of the abdomen. You detect ascites on clinical examination and perform a paracentesis. The results show a WBC count of 300 leukocytes/µL with 35% polymorphonuclear cells. The peritoneal albumin level is 1.2 g/dL, protein is 2.0 g/dL, and triglycerides are 320 mg/dL. Peritoneal cultures are pending. Serum albumin is 2.6 g/dL. Which of the following is the most likely diagnosis?
++
++
++
A. Congestive heart failure
++
++
B. Peritoneal tuberculosis
++
++
C. Peritoneal carcinomatosis
++
++
++
++
+
++
The answer is A. (Chaps. 59 and 357) Diagnostic paracentesis is part of the routine evaluation in a patient with ascites. Fluid should be examined for its gross appearance, protein content, cell count and differential, and albumin. Cytologic and culture studies should be performed when one suspects infection or malignancy. The serum-ascites albumin gradient (SAG) offers the best correlation with portal pressure. A high gradient (>1.1 g/dL) is characteristic of uncomplicated cirrhotic ascites and differentiates ascites caused by portal hypertension from ascites not caused by portal hypertension in more than 95% of cases. Conditions that cause a low gradient include more “exudative” processes such as infection, malignancy, and inflammatory processes. Similarly, congestive heart failure and nephrotic syndrome cause high gradients. In this patient, the SAG is 1.5 g/dL, indicating a high gradient. The low number of leukocytes and polymorphonuclear cells makes bacterial or tubercular infection unlikely. Chylous ascites often is characterized by an opaque milky fluid with a triglyceride level greater than 1000 mg/dL in addition to a low SAG.
++
++
++
Which of the following is the most common symptom or sign of liver disease?
++
++
++
++
++
++
++
++
++
++
++
E. Right upper quadrant pain
+
++
The answer is A. (Chap. 357) The most common and most characteristic symptom of liver disease is fatigue. Unfortunately, it is also very nonspecific, with little specific diagnostic utility. The fatigue in liver disease seems to improve in the morning and worsen throughout the day, but it can be intermittent. Jaundice is the hallmark of liver disease and is much more specific. Jaundice, however, is typically a sign of more advanced disease. Itching is also typically a symptom of more advanced disease and is more common in cholestatic causes of liver disease. Nausea often occurs in severe disease and can be accompanied by vomiting. Right upper quadrant pain is a less common symptom and indicates stretching of the liver capsule.
++
++
++
In women, what is the average amount of reported daily alcohol intake that is associated with the development of chronic liver disease?
++
++
++
++
++
++
++
++
++
++
++
+
++
The answer is B. (Chap. 357) Women are more susceptible to the effects of alcohol on the liver. On average, drinking about two drinks daily can lead to chronic liver disease in women, whereas men report drinking about three drinks daily. However, in individuals with alcoholic cirrhosis, the average daily alcohol intake is usually much higher, and heavy levels of drinking for more than 10 years are typical prior to the onset of liver disease.
++
++
++
All of the following are CAGE questions, which should be a component of the medical history focusing on alcohol abuse and dependence, EXCEPT:
++
++
++
A. Do you feel like you have a greater tolerance for alcohol than your friends?
++
++
B. Have you ever felt you ought to cut down on your drinking?
++
++
C. Have people annoyed you by criticizing your drinking?
++
++
D. Have you ever felt guilty or bad about your drinking?
++
++
E. Have you ever had a drink first thing in the morning to steady your nerves or get rid of a hangover?
+
++
The answer is A. (Chap. 357) In assessing alcohol intake, the history should also focus on whether alcohol abuse or dependence is present. Alcoholism is usually defined by the behavioral patterns and consequences of alcohol intake, not by the amount. Abuse is defined by a repetitive pattern of drinking alcohol that has adverse effects on social, family, occupational, or health status. Dependence is defined by alcohol-seeking behavior, despite its adverse effects. Many alcoholics demonstrate both dependence and abuse, and dependence is considered the more serious and advanced form of alcoholism. A clinically helpful approach to diagnosis of alcohol dependence and abuse is the use of the CAGE questionnaire, which is recommended for all medical history taking. One “yes” response should raise suspicion of an alcohol use problem, and more than one “yes” response is a strong indication of abuse or dependence.
++
++
++
Elevation in all of the following laboratory studies would be indicative of liver disease EXCEPT:
++
++
++
++
++
B. Aspartate aminotransferase
++
++
++
++
D. Unconjugated bilirubin
++
++
+
++
The answer is D. (Chap. 358) It is important to understand the patterns of laboratory abnormalities that indicate liver disease is present. One way to consider laboratory evaluation of liver disease is to consider three general categories of tests: tests based on excretory function of the liver, tests of biosynthetic activity of the liver, and coagulation factors. The most common tests of liver function fall under the category of tests based on the detoxification and excretory function of the liver. These include serum bilirubin, urine bilirubin, ammonia, and enzyme levels. Bilirubin can exist as a conjugated and an unconjugated form. The unconjugated form is often referred to as the indirect fraction. Isolated elevation in the unconjugated form of bilirubin is typically not related to liver disease but is most commonly seen in hemolysis and a number of benign genetic conditions such as Gilbert syndrome. In contrast, conjugated hyperbilirubinemia almost always indicates disease of the liver or biliary tract. Conjugated bilirubin is water soluble and excreted in the urine, but unconjugated bilirubin is not. Rather, it binds to albumin in the blood. Therefore, bilirubinuria implies liver disease as well. Among the serum enzymes, it is useful to consider enzymes as those that are associated with hepatocellular injury or those that reflect cholestasis. Alanine and aspartate aminotransferases are the primary enzymes that indicate hepatocyte injury. Alkaline phosphatase is the most common enzyme elevated in cholestasis, but bone disease also causes increased alkaline phosphatase. In some cases, one needs additional information to determine whether the alkaline phosphatase is liver or bone in origin. Other tests that would be elevated in cholestatic liver disease are 5′-nucleotidase and γ-glutamyl transferase. The primary test of synthetic function is measurement of serum albumin. Coagulation factors can be directly measured, but impaired production of coagulation factors in liver disease is primarily inferred from elevations in prothrombin time.
++
++
++
All of the following statements regarding liver function tests are true EXCEPT:
++
++
++
A. Alanine aminotransferase (ALT) is found in liver, cardiac muscle, skeletal muscle, and kidney.
++
++
B. Elevation of aspartate aminotransferase (AST) and ALT to >1000 IU/L is typical of ischemic hepatitis.
++
++
C. Elevation of AST is more specific for liver dysfunction than elevation of ALT.
++
++
D. Increased AST and ALT with an AST:ALT ratio of >3 is typical of acute viral hepatitis.
++
++
E. The magnitude of elevated AST and ALT has important prognostic significance in acute hepatitis.
+
++
The answer is B. (Chap. 358) The aminotransferases are sensitive indicators of liver cell injury and are most helpful in recognizing acute hepatocellular diseases such as hepatitis. They include aspartate aminotransferase (AST) and alanine aminotransferase (ALT). AST is found in the liver, cardiac muscle, skeletal muscle, kidneys, brain, pancreas, lungs, leukocytes, and erythrocytes in decreasing order of concentration. ALT is found primarily in the liver and is therefore a more specific indicator of liver injury. The aminotransferases are normally present in the serum in low concentrations. These enzymes are released into the blood in greater amounts when there is damage to the liver cell membrane, resulting in increased permeability. Liver cell necrosis is not required for the release of the aminotransferases, and there is a poor correlation between the degree of liver cell damage and the level of the aminotransferases. Thus, the absolute elevation of the aminotransferases is of no prognostic significance in acute hepatocellular disorders.
++
Any type of liver cell injury can cause modest elevations in the serum aminotransferases. Levels of up to 300 IU/L are nonspecific and may be found in any type of liver disorder. Minimal ALT elevations in asymptomatic blood donors rarely indicate severe liver disease; studies have shown that fatty liver disease is the most likely explanation. Striking elevations—i.e., aminotransferases >1000 IU/L—occur almost exclusively in disorders associated with extensive hepatocellular injury such as viral hepatitis, ischemic liver injury (prolonged hypotension or acute heart failure), or toxin- or drug-induced liver injury.
++
The pattern of the aminotransferase elevation can be helpful diagnostically. In most acute hepatocellular disorders, the ALT is higher than or equal to the AST. Whereas the AST:ALT ratio is typically <1 in patients with chronic viral hepatitis and nonalcoholic fatty liver disease, a number of groups have noted that as cirrhosis develops, this ratio rises to >1. An AST:ALT ratio >2:1 is suggestive, whereas a ratio >3:1 is highly suggestive, of alcoholic liver disease. The AST in alcoholic liver disease is rarely >300 IU/L, and the ALT is often normal. The aminotransferases are usually not greatly elevated in obstructive jaundice.
++
++
++
A 26-year-old male resident is noticed by his attending physician to have yellow eyes after his 24-hour call period. When asked, the resident states he has no medical history, but on occasion, he has thought he might have mild jaundice when he is stressed or has more than four to five alcoholic drinks. He never sought medical treatment because he was uncertain, and his eyes would return fully to normal within 2 days. He denies nausea, abdominal pain, dark urine, light-colored stools, pruritus, or weight loss. On examination, he has a BMI of 20.1 kg/m2, and his vital signs are normal. Scleral icterus is present. There are no stigmata of chronic liver disease. The patient’s abdomen is soft and nontender. The liver span is 8 cm to percussion. The liver edge is smooth and palpable only with deep inspiration. The spleen is not palpable. Laboratory examinations are normal except for a total bilirubin of 3.0 mg/dL. Direct bilirubin is 0.2 mg/dL. AST, ALT, and alkaline phosphatase are normal. Hematocrit, lactate dehydrogenase, and haptoglobin are normal. Which of the following is the most likely diagnosis?
++
++
++
A. Autoimmune hemolytic anemia
++
++
B. Crigler-Najjar syndrome type 1
++
++
++
++
D. Dubin-Johnson syndrome
++
++
+
++
The answer is E. (Chap. 359) This patient is presenting with an asymptomatic and mild elevation in unconjugated hyperbilirubinemia that has occurred during a time of increased stress, fatigue, and likely decreased caloric intake. This presentation is characteristic of Gilbert syndrome, an inherited disorder of bilirubin conjugation. In Gilbert syndrome, there is a mutation of the UGT1A1 gene that encodes bilirubin UDP-glucuronosyltransferase, which leads to a reduction in activity on the enzyme to 10%–35% of normal. This enzyme is of critical importance in the conjugation of bilirubin. Most of the time, there is no apparent jaundice because the reduced ability to conjugate bilirubin is not reduced to a degree that leads to an elevation of bilirubin. However, during times of stress, fatigue, alcohol use, decreased caloric intake, or intercurrent illness, the enzyme can become overwhelmed, leading to a mild hyperbilirubinemia. Typical bilirubin levels are less than 4.0 mg/dL unless the individual is ill or fasting. Diagnosis usually occurs during young adulthood, and episodes are self-limited and benign. If a liver biopsy were to be performed, hepatic histology would be normal. No treatment is necessary because there are no long-term consequences of Gilbert syndrome, and patient reassurance is recommended. Other inherited disorders of bilirubin conjugation are Crigler-Najjar syndromes types 1 and 2. Crigler-Najjar syndrome type 1 is a congenital disease characterized by more dramatic elevations in bilirubin as high as 20–45 mg/dL that is first diagnosed in the neonatal period and is present throughout life. This rare disorder was once fatal in early childhood due to the development of kernicterus. However, with phototherapy, individuals are now able to survive into adulthood, although neurologic deficits are common. Crigler-Najjar syndrome type 2 is similar to type 1, but the elevations in bilirubin are not as great. Kernicterus is rare. This is due to the fact that there is some residual function of the bilirubin UDP-glucuronosyltransferase enzyme (<10%) that is totally absent in type 1 disease. Hemolysis is another frequent cause of elevated unconjugated bilirubin. Hemolysis can be caused by many factors including medications, autoimmune disorders, and inherited disorders, among others. However, the normal hematocrit, lactate dehydrogenase, and haptoglobin eliminate hemolysis as a possibility. Dubin-Johnson syndrome is another congenital hyperbilirubinemia. However, it is a predominantly conjugated hyperbilirubinemia caused by a defect in biliary excretion from hepatocytes. Obstructive choledocholithiasis is characterized by right upper quadrant pain that is often exacerbated by fatty meals. The absence of symptoms or elevation in other liver function tests, especially alkaline phosphatase, also makes this diagnosis unlikely.
++
++
++
A 26-year-old male resident is noticed by his attending physician to have yellow eyes after his 24-hour call period. When asked, the resident states he has no medical history, but on occasion, he has thought he might have mild jaundice when he is stressed or has more than four to five alcoholic drinks. He never sought medical treatment because he was uncertain, and his eyes would return fully to normal within 2 days. He denies nausea, abdominal pain, dark urine, light-colored stools, pruritus, or weight loss. On examination, he has a BMI of 20.1 kg/m2, and his vital signs are normal. Scleral icterus is present. There are no stigmata of chronic liver disease. The patient’s abdomen is soft and nontender. The liver span is 8 cm to percussion. The liver edge is smooth and palpable only with deep inspiration. The spleen is not palpable. Laboratory examinations are normal except for a total bilirubin of 3.0 mg/dL. Direct bilirubin is 0.2 mg/dL. AST, ALT, and alkaline phosphatase are normal. Hematocrit, lactate dehydrogenase, and haptoglobin are normal. What is the next step in the evaluation and management of the patient?
++
++
++
++
++
B. Peripheral blood smear
++
++
++
++
++
++
E. Right upper quadrant ultrasound
+
++
The answer is D. (Chap. 359) This patient is presenting with an asymptomatic and mild elevation in unconjugated hyperbilirubinemia that has occurred during a time of increased stress, fatigue, and likely decreased caloric intake. This presentation is characteristic of Gilbert syndrome, an inherited disorder of bilirubin conjugation. In Gilbert syndrome, there is a mutation of the UGT1A1 gene that encodes bilirubin UDP-glucuronosyltransferase, which leads to a reduction in activity on the enzyme to 10%–35% of normal. This enzyme is of critical importance in the conjugation of bilirubin. Most of the time, there is no apparent jaundice because the reduced ability to conjugate bilirubin is not reduced to a degree that leads to an elevation of bilirubin. However, during times of stress, fatigue, alcohol use, decreased caloric intake, or intercurrent illness, the enzyme can become overwhelmed, leading to a mild hyperbilirubinemia. Typical bilirubin levels are less than 4.0 mg/dL unless the individual is ill or fasting. Diagnosis usually occurs during young adulthood, and episodes are self-limited and benign. If a liver biopsy were to be performed, hepatic histology would be normal. No treatment is necessary because there are no long-term consequences of Gilbert syndrome, and patient reassurance is recommended. Other inherited disorders of bilirubin conjugation are Crigler-Najjar syndromes types 1 and 2. Crigler-Najjar syndrome type 1 is a congenital disease characterized by more dramatic elevations in bilirubin as high as 20–45 mg/dL that is first diagnosed in the neonatal period and is present throughout life. This rare disorder was once fatal in early childhood due to the development of kernicterus. However, with phototherapy, individuals are now able to survive into adulthood, although neurologic deficits are common. Crigler-Najjar syndrome type 2 is similar to type 1, but the elevations in bilirubin are not as great. Kernicterus is rare. This is due to the fact that there is some residual function of the bilirubin UDP-glucuronosyltransferase enzyme (<10%) that is totally absent in type 1 disease. Hemolysis is another frequent cause of elevated unconjugated bilirubin. Hemolysis can be caused by many factors including medications, autoimmune disorders, and inherited disorders, among others. However, the normal hematocrit, lactate dehydrogenase, and haptoglobin eliminate hemolysis as a possibility. Dubin-Johnson syndrome is another congenital hyperbilirubinemia. However, it is a predominantly conjugated hyperbilirubinemia caused by a defect in biliary excretion from hepatocytes. Obstructive choledocholithiasis is characterized by right upper quadrant pain that is often exacerbated by fatty meals. The absence of symptoms or elevation in other liver function tests, especially alkaline phosphatase, also makes this diagnosis unlikely.
++
++
++
Which of the following statements regarding the hyperbilirubinemia seen in patients with significant intravascular hemolysis is true?
++
++
++
A. Bilirubin values <4 mg/dL imply concomitant gallbladder or biliary dysfunction.
++
++
B. Bilirubin values >4 mg/dL (68 μmol/L) imply concomitant liver dysfunction.
++
++
C. It is typically composed of 50% conjugated and 50% unconjugated bilirubin.
++
++
D. Prolonged hemolysis may result in the development of nephrolithiasis due to bile pigment stones.
+
++
The answer is B. (Chap. 359) Increased destruction of erythrocytes leads to increased bilirubin turnover and unconjugated hyperbilirubinemia; the hyperbilirubinemia is usually modest in the presence of normal liver function. Hemolysis alone cannot result in a sustained hyperbilirubinemia of more than 4 mg/dL (~68 μmol). Higher values imply concomitant hepatic dysfunction. When hemolysis is the only abnormality in an otherwise healthy individual, the result is a purely unconjugated hyperbilirubinemia, with the direct-reacting fraction as measured in a typical clinical laboratory being ≤15% of the total serum bilirubin. In the presence of systemic disease, which may include a degree of hepatic dysfunction, hemolysis may produce a component of conjugated hyperbilirubinemia in addition to an elevated unconjugated bilirubin concentration. Prolonged hemolysis may lead to the precipitation of bilirubin salts within the gallbladder or biliary tree, resulting in the formation of gallstones in which bilirubin, rather than cholesterol, is the major component. Such pigment stones may lead to acute or chronic cholecystitis, biliary obstruction, or any other biliary tract consequence of calculous disease.
++
++
++
A 34-year-old man presents to the physician complaining of yellow eyes. For the past week, he has felt ill, with decreased oral intake, low-grade fevers (~100°F), fatigue, nausea, and occasional vomiting. With the onset of jaundice, he has noticed pain in his right upper quadrant. He currently uses marijuana and ecstasy and has a prior history of injection drug use with cocaine. He has no other past medical history, but he was unable to donate blood for reasons that he cannot recall 4 years previously. His social history is remarkable for working as a veterinary assistant. On sexual history, he reports five male sexual partners over the past 6 months. He does not consistently use condoms. On physical examination, he appears ill and has obvious jaundice with scleral icterus. His liver is 15 cm to percussion, palpable 6 cm below the right costal margin. The edge is smooth and tender to palpation. The spleen is not enlarged. There are no stigmata of chronic liver disease. His AST is 1232 IU/L, ALT is 1560 IU/L, alkaline phosphatase is 394 IU/L, total bilirubin is 13.4 mg/dL, and direct bilirubin is 12.2 mg/dL. His international normalized ratio (INR) is 2.3, and activated partial thromboplastin time (aPTT) is 52 seconds. Hepatitis serologies are sent and reveal the following
++
++
What is the cause of the patient’s current clinical presentation?
++
++
++
A. Acute hepatitis A infection
++
++
B. Acute hepatitis B infection
++
++
C. Acute hepatitis C infection
++
++
D. Chronic hepatitis B infection
++
++
E. Drug-induced hepatitis
+
++
The answer is B. (Chap. 360) This patient presents with acute hepatitis, which has numerous etiologies. These include viruses, toxins/drugs, autoimmune diseases, metabolic disease, alcohol, ischemia, pregnancy, and other infectious etiologies including rickettsial diseases and leptospirosis. In this clinical scenario, the patient does have risk factors for hepatitis A, B, and C infection, including being a man who has sex with men and a prior history of injection drug use. All acute viral hepatitis presents with a similar clinical pattern, although incubation periods vary after exposure. The most common initial symptoms are fatigue, anorexia, nausea, vomiting, myalgias, and headache. These symptoms precede the onset of jaundice by about 1–2 weeks. Once jaundice develops, the prodromal symptoms regress. On physical examination, there is usually obvious icterus with an enlarged and tender liver. Splenomegaly can occur. AST and ALT are elevated with peak levels that are quite variable between 400 and 4000 IU/L, and alkaline phosphatase levels are increased to a much lesser degree. Hyperbilirubinemia (levels 5–20 mg/dL) occurs with primarily increased levels of conjugated bilirubin. Thus, it is important to recognize the patterns of antibody production in the viral hepatitides. Hepatitis A is an RNA virus that presents with acute hepatitis and is transmitted by the fecal-oral route. In the acute state, the immunoglobulin (Ig) M would be elevated, which is not seen in this scenario. Hepatitis B virus is a DNA virus with three common antigens that are tested serologically to determine the time course of the illness. These antigens are the surface antigen, the core antigen, and the e antigen, which is a nucleocapsid protein produced from the same gene as the core antigen but immunologically distinct. Several distinct patterns can be observed. In acute hepatitis B, the core IgM, surface antigen, and e antigen are all positive, which is what is seen in this case. At this point, the patient is highly infectious, with viral shedding in body fluids, including saliva. In a late acute infection, core IgG may be positive at the same time as surface and e antigen positivity. In chronic hepatitis B, this same pattern of serologies is seen. If a patient has a prior infection without development of chronic hepatitis, the core IgG and surface antibody is positive. However, when immunity is obtained via vaccination, only the surface antibody (SAb) is positive, the e antigen and surface antigen will be negative since the patient was never infected. The variety of antigen-antibody positivities that can result are outlined in Table VIII-51 below. Acute hepatitis C often is detectable with contemporary immunoassays early during the disease when the aminotransferases are positive. Thus, a positive HCV antibody could indicate acute hepatitis C in this individual. However, given his clinical history of prior injection drug use and inability to donate blood, this likely indicates chronic hepatitis C infection. In some instances, ecstasy has been reported to cause drug-induced hepatitis, but given the viral serologies in this patient, this would be unlikely.
++
++
++
++
A 34-year-old man presents to the physician complaining of yellow eyes. For the past week, he has felt ill, with decreased oral intake, low-grade fevers (~100°F), fatigue, nausea, and occasional vomiting. With the onset of jaundice, he has noticed pain in his right upper quadrant. He currently uses marijuana and ecstasy and has a prior history of injection drug use with cocaine. He has no other past medical history, but he was unable to donate blood for reasons that he cannot recall 4 years previously. His social history is remarkable for working as a veterinary assistant. On sexual history, he reports five male sexual partners over the past 6 months. He does not consistently use condoms. On physical examination, he appears ill and has obvious jaundice with scleral icterus. His liver is 15 cm to percussion, palpable 6 cm below the right costal margin. The edge is smooth and tender to palpation. The spleen is not enlarged. There are no stigmata of chronic liver disease. His AST is 1232 IU/L, ALT is 1560 IU/L, alkaline phosphatase is 394 IU/L, total bilirubin is 13.4 mg/dL, and direct bilirubin is 12.2 mg/dL. His international normalized ratio (INR) is 2.3, and activated partial thromboplastin time (aPTT) is 52 seconds. Hepatitis serologies are sent and reveal the following
++
++
In the patient described, what would be the best approach to prevent development of chronic hepatitis?
++
++
++
A. Administration of anti-hepatitis A virus immunoglobulin (Ig) G
++
++
B. Administration of lamivudine
++
++
C. Administration of pegylated interferon-α plus ribavirin
++
++
D. Administration of prednisone beginning at a dose of 1 mg/kg daily
++
++
E. Do nothing and observe as 99% of individuals with this disease recover
+
++
The answer is E. (Chap. 360) No treatment is recommended for acute hepatitis B in most individuals because 99% of infected individual recover without assistance. Therefore, it would not be expected that an individual would derive any particular benefit from treatment. In severe acute hepatitis B, nucleoside analogues, including lamivudine, have been used successfully, although there are no clinical trial data to support such an approach. Hepatitis A is an acute and self-limited illness that does not progress to chronic liver disease. Thus, no treatment is required. Anti–hepatitis A virus immunoglobulin can be give prophylactically following a known exposure to prevent development of disease, but it is not helpful in established disease. There is no role for oral or intravenous corticosteroids in the treatment of acute viral hepatitis of any etiology. It has demonstrated no clinical benefit and may increase the risk of developing chronic disease.
++
++
++
Which of the following viral causes of acute hepatitis is most likely to cause fulminant hepatitis in a pregnant woman?
++
++
++
++
++
++
++
++
++
++
++
+
++
The answer is E. (Chap. 360) In most instances, patients with any form of acute viral hepatitis do not succumb to fulminant liver failure. However, pregnant women are highly susceptible to fulminant hepatic failure in the setting of acute hepatitis E infection. This RNA virus is an enteric virus that is endemic in India, Asia, Africa, the Middle East, and Central America and is spread via contaminated water supplies. Person-to-person spread is rare. Generally, the clinical course of hepatitis E infection is mild and the rate of fulminant hepatitis is only 1%–2%. However, in pregnant women, this is as high as 10%–20%. For hepatitis A and C, the rate of fulminant hepatic failure is about 0.1% or less. It is slightly higher for hepatitis B, at around 0.1%–1%. Hepatitis D occurs as a coinfection with hepatitis B virus. When the two viruses are acquired simultaneously, the rate of fulminant hepatitis is about 5% or less. When hepatitis D is acquired in the setting of chronic hepatitis B infection, this number rises to 20%.
++
++
++
A 16-year-old girl had visited your clinic 1 month ago with jaundice, vomiting, malaise, and anorexia. Two other family members were ill with similar symptoms. Based on viral serologies, including a positive anti-hepatitis A virus IgM, a diagnosis of hepatitis A was made. The patient was treated conservatively, and 1 week after first presenting, she appeared to have made a full recovery. She returns to your clinic today complaining of the same symptoms she had 1 month ago. She is jaundiced, and an initial panel of laboratory tests returns elevated transaminases. Which of the following offers the best explanation of what has occurred in this patient?
++
++
++
A. Co-infection with hepatitis C
++
++
B. Inappropriate treatment of initial infection
++
++
C. Incorrect initial diagnosis; this patient likely has hepatitis B
++
++
D. Reinfection with hepatitis A
++
++
E. Relapse of hepatitis A
+
++
The answer is E. (Chap. 360) Hepatitis A virus (HAV) is an acute, self-limited virus that is acquired almost exclusively via the fecal-oral route. It is classically a disease of poor hygiene and overcrowding. Outbreaks have been traced to contaminated water, milk, frozen raspberries and strawberries, green onions, and shellfish. Infection occurs mostly in children and young adults. It almost invariably resolves spontaneously and results in lifelong immunity. Fulminant disease occurs in ≤0.1% of cases, and there is no chronic form (in contrast to hepatitis B and C). Diagnosis is made by demonstrating a positive IgM antibody to HAV, as described earlier. An IgG antibody to HAV indicates immunity, obtained by previous infection or vaccination. A small proportion of patients will experience relapsing hepatitis weeks to months after a full recovery from HAV infection. This too is self-limited. There is no approved antiviral therapy for HAV. An inactivated vaccine has decreased the incidence of the disease, and it is recommended for all U.S. children, for high-risk adults, and for travelers to endemic areas. Passive immunization with immunoglobulin is also available, and it is effective in preventing clinical disease before exposure or during the early incubation period.
++
++
++
A 26-year-old woman presents to your clinic and is interested in getting pregnant. She seeks your advice regarding vaccines she should obtain, and in particular asks about the hepatitis B vaccine. She works as a receptionist for a local business, denies alcohol or illicit drug use, and is in a monogamous relationship. Which of the following is true regarding hepatitis B vaccination?
++
++
++
A. Hepatitis B vaccine consists of two intramuscular doses 1 month apart.
++
++
B. Only patients with defined risk factors need to be vaccinated.
++
++
C. Pregnancy is not a contraindication to the hepatitis B vaccine.
++
++
D. This patient’s hepatitis serologies should be checked prior to vaccination.
++
++
E. Vaccination should not be administered to children under 2 years old.
+
++
The answer is C. (Chap. 360) The current hepatitis B vaccine is a recombinant vaccine consisting of yeast-derived hepatitis B surface antigen particles. A strategy of vaccinating only high-risk individuals in the United States has been shown to be ineffective, and universal vaccination against hepatitis B is now recommended. Pregnancy is not a contraindication to vaccination. Vaccination should ideally be performed in infancy. Routine evaluation of hepatitis serologies is not cost-effective and is not recommended. The vaccine is given in three divided intramuscular doses at 0, 1, and 6 months.
++
++
++
An 18-year-old man presents to a rural clinic with nausea, vomiting, anorexia, abdominal discomfort, myalgias, and jaundice. He describes occasional alcohol use and is sexually active. He describes using heroin and cocaine “a few times in the past.” He works as a short-order cook in a local restaurant. He has lost 15.5 kg (34 lb) since his last visit to clinic and appears emaciated and ill. On examination, he is noted to have icteric sclerae and a palpable, tender liver below the right costal margin. In regard to acute hepatitis, which of the following is true?
++
++
++
A. A distinction between viral etiologies cannot be made using clinical criteria alone.
++
++
B. Based on age and risk factors, he is likely to have hepatitis B infection.
++
++
C. He does not have hepatitis E virus, as this infects only pregnant women.
++
++
D. This patient cannot have hepatitis C because his presentation is too acute.
++
++
E. This patient does not have hepatitis A because his presentation is too fulminant.
+
++
The answer is A. (Chap. 360) A clear distinction between viral etiologies of acute hepatitis cannot be made on clinical or epidemiologic features alone. This patient is at risk of many forms of hepatitis due to his lifestyle. Given his occupation in food services, from a public health perspective, it is important to make an accurate diagnosis. Serologies must be obtained to make a diagnosis. Although hepatitis C virus typically does not present as an acute hepatitis, this is not absolute. Hepatitis E virus infects men and women equally and resembles hepatitis A virus in clinical presentation. This patient should be questioned regarding IV drug use, and in addition to hepatitis serologies, an HIV test should be performed.
++
++
++
A 36-year-old man presents with fatigue and tea-colored urine for 5 days. Physical examination reveals jaundice and tender hepatomegaly but is otherwise unremarkable. Laboratories are remarkable for an AST of 2400 IU/L and an ALT of 2640 IU/L. Alkaline phosphatase is 210 IU/L. Total bilirubin is 8.6 mg/dL. Which of the following diagnoses is least likely to cause this clinical picture and these laboratory abnormalities?
++
++
++
A. Acute hepatitis A infection
++
++
B. Acute hepatitis B infection
++
++
C. Acute hepatitis C infection
++
++
D. Acetaminophen ingestion
++
++
+
++
The answer is C. (Chaps. 358 and 360) Causes of extreme elevations in serum transaminases generally fall into a few major categories, including viral infections, toxic ingestions, and vascular/hemodynamic causes. Both acute hepatitis A and hepatitis B infections may be characterized by high transaminases. Fulminant hepatic failure may occur, particularly in situations in which acute hepatitis A occurs on top of chronic hepatitis C infection or if hepatitis B and hepatitis D are co-transmitted. Most cases of acute hepatitis A or B infection in adults are self-limited. Hepatitis C is an RNA virus that does not typically cause acute hepatitis. However, it is associated with a high probability of chronic infection. Therefore, progression to cirrhosis and hepatoma is increased in patients with chronic hepatitis C infection. Extreme transaminitis is highly unlikely with acute hepatitis C infection. Acetaminophen remains one of the major causes of fulminant hepatic failure and is managed by prompt administration of N-acetylcysteine. Budd-Chiari syndrome is characterized by posthepatic thrombus formation. It often presents with jaundice, painful hepatomegaly, ascites, and elevated transaminases.
++
++
++
Which of the following drugs has a direct toxic effect on hepatocytes?
++
++
++
++
++
++
++
++
++
++
++
+
++
The answer is A. (Chap. 361) The liver is the primary site for metabolism of many drugs and, as such, is susceptible to injury related to drugs and toxins. Indeed, the most common cause of acute hepatic failure is drug-induced liver injury. In general, it is useful to think of chemical hepatotoxicity within two broad categories: direct toxic effects or idiosyncratic reactions. Drugs or toxins that cause a direct toxic effect on liver are either poisons themselves or are metabolized to toxic substances. With agents that cause a direct toxic effect on hepatocytes, there is a predictable, dose-related pattern of injury, and the time to effect is relatively short. The most common drug or toxin causing direct hepatocyte toxicity is acetaminophen. In therapeutic doses, acetaminophen does not cause liver injury. However, in higher doses, one of the metabolites of acetaminophen, N-acetyl-p-benzoquinone imine (NAPQI), can overwhelm the glutathione stores of the liver that are necessary to convert NAPQI to a nontoxic metabolite and lead to hepatocyte necrosis. Other medications or toxins that cause direct hepatocyte injury are carbon tetrachloride, trichloroethylene, tetracycline, and the Amanita phalloides mushroom. More commonly known as the death cap mushroom, ingestion of a single A phalloides mushroom can contain enough hepatotoxin to be lethal. Idiosyncratic reactions are infrequent and unpredictable. There is no dose dependency, and the timing of hepatic injury has little association with the duration of drug treatment. Many drugs produce idiosyncratic reactions, and often, it is difficult to know when an idiosyncratic reaction will lead to more serious liver failure. Often, mild increases in transaminase levels will occur, but over time, adaptation leads to a return of liver enzymes to normal levels. In other instances, idiosyncratic reactions can lead to fulminant hepatic failure. Although rare, serious hepatic reactions can lead to medications being removed from the market. It is now recognized that many idiosyncratic reactions are related to metabolites that cause liver injury. However, it is likely that individual genetic variations in liver metabolism are the primary cause, and these are not predictable effects of the drug given our current state of knowledge. Common medications that can lead to idiosyncratic drug reactions include halothane, isothane, isoniazid, 3-hydroxy-3-methylglutaryl–coenzyme A (HMG-CoA) reductase inhibitors, and chlorpromazine.
++
++
++
A 32-year-old woman is admitted to the ICU following an overdose of acetaminophen with co-ingestion of alcohol. She was known to be alert and interactive about 4 hours prior to her presentation when she had a fight with her boyfriend who then left the home. When he returned 6 hours later, he found an empty bottle of acetaminophen 500-mg capsules as well as an empty vodka bottle. The exact number of pills in the bottle is unknown, but the full bottle held as much as 50 capsules. The patient was unresponsive and had vomited, so her boyfriend called 911. Upon arrival to the emergency department, the patient is stuporous. Her vital signs are: pulse 109 bpm, respiratory rate 20 breaths/min, blood pressure 96/52 mmHg, and oxygen saturation 95% on room air. Her examination shows mild nonspecific abdominal pain with palpation. The liver is not enlarged. Her initial laboratory values show a normal complete blood count, normal electrolytes, and kidney function. The AST is 68 IU/L, ALT is 46 IU/L, alkaline phosphatase is 110 IU/L, and total bilirubin is 1.2 mg/dL. Glucose and coagulation studies are normal. The serum alcohol level is 210 g/dL. The acetaminophen level is 350 μg/mL. What is the most appropriate next step in the treatment of this patient?
++
++
++
A. Administration of activated charcoal or cholestyramine
++
++
B. Administration of N-acetylcysteine 140 mg/kg followed by 70 mg/kg every 4 hours for a total of 15–20 doses
++
++
C. Continued monitoring of liver function, glucose, and coagulation studies every 4 hours with administration of N-acetylcysteine if these begin to change
++
++
D. Do nothing as normal liver function tests and coagulation studies are indicative of only a minor ingestion
++
++
E. Initiate hemodialysis for toxin clearance
+
++
The answer is B. (Chap. 305) Acetaminophen overdose is the most common cause of acute liver failure and the most common cause of drug-induced liver failure that leads to transplantation. Acetaminophen is metabolized in the liver through two pathways. The primary pathway is a phase II reaction that produces nontoxic sulfate and glucuronide metabolites. The minor pathway occurs through a phase I reaction leading to production of NAPQI. This metabolite is directly toxic to liver cells and can lead to hepatocyte necrosis. With therapeutic use of acetaminophen, glutathione in the liver rapidly converts NAPQI to a nontoxic metabolite that is excreted in the urine. However, glutathione stores can become depleted in the setting of a large acute ingestion, chronic alcoholism, or chronic ingestion of increased acetaminophen. In addition, alcohol upregulates the first enzyme in the metabolic pathway, causing NAPQI to accumulate more quickly in alcoholics. Given the known hepatotoxicity of acetaminophen, the U.S. Food and Drug Administration has recommended a maximum daily dose of no more than 3.25 g, with lower doses in individuals with chronic alcohol use. Acute ingestions of 10–15 g of acetaminophen are sufficient to cause clinical evidence of liver injury, and doses higher than 25 g can lead to fatal hepatic necrosis. The course of illness with acute acetaminophen ingestion follows a predictable pattern. Nausea, vomiting, abdominal pain, and shock occur within 4–12 hours after ingestion. Liver enzymes and synthetic function are normal at this time. Within 24–48 hours, these symptoms subside and are followed by evidence of hepatic injury. Maximal levels of aminotransferases can reach more than 10,000 IU/L and may not occur until 4–6 days after ingestion. These patients must be followed carefully for fulminant hepatic failure with serious complications including encephalopathy, cerebral edema, marked coagulopathy, renal failure, metabolic acidosis, electrolyte abnormalities, and refractory shock. Levels of acetaminophen are predictive of the development of hepatotoxicity. The first level should be measured no sooner than 4 hours after a known ingestion. Levels should be plotted on a nomogram that relates levels to the time after ingestion. If, at 4 hours, the acetaminophen level is greater than 300 μg/mL, significant hepatotoxicity is likely. In the setting of overdose, it may be difficult to know the exact quantity and timing of an ingestion. For the patient presenting in the clinical scenario in this question, her level of greater than 300 μg/mL is quite concerning for a large ingestion, and treatment should be initiated immediately. The primary treatment for acetaminophen overdose is N-acetylcysteine. N-Acetylcysteine acts to replete glutathione levels in the liver and also provides a reservoir of sulfhydryl groups to bind to the toxic metabolites. The typical regimen of N-acetylcysteine is 140 mg/kg given as a loading dose, followed by 70 mg/kg every 4 hours for a total of 15–20 doses. This drug can also be given by continuous infusion. Activated charcoal or cholestyramine should only be given if the patient presents within 30 minutes after ingestion. Hemodialysis will not accelerate clearance of acetaminophen and will not protect the liver. Most patients with fulminant hepatic failure develop acute renal failure, often requiring hemodialysis. If a patient survives an acetaminophen overdose, there is usually no chronic liver injury.
++
++
++
A 31-year-old healthcare worker is found to have a newly positive tuberculin skin test 6 weeks after an exposure to a patient with active pulmonary tuberculosis. He is asymptomatic and has a normal chest radiograph. Which of the following statements regarding initiation of isoniazid (INH) prophylactic therapy is true?
++
++
++
A. Acute hepatocellular injury due to INH is an idiosyncratic reaction that will manifest within the first 2 months of initiation of therapy.
++
++
B. Controlled trials have demonstrated that monthly monitoring of aminotransferase levels reduces morbidity in U.S. healthcare workers receiving INH prophylaxis.
++
++
C. Elevation of aminotransferase levels in the first 2 months of therapy is an indication to stop INH and switch to another drug.
++
++
D. The patient has a 50%–70% chance of transient elevation of his aminotransferase levels in the first 2 months of treatment.
++
++
E. The frequency of acute hepatocellular injury due to INH is age-dependent, increasing in patients >35 years of age.
+
++
The answer is E. (Chap. 361) Isoniazid (INH) remains central to most antituberculous prophylactic and therapeutic regimens, despite its long-standing recognition as a hepatotoxin. In 10% of patients treated with INH, elevated serum aminotransferase levels develop during the first few weeks of therapy; however, these elevations in most cases are self-limited, mild (values for ALT <200 IU/L), and resolve despite continued drug use. This adaptive response allows continuation of the agent if symptoms and progressive enzyme elevations do not follow the initial elevations. Acute hepatocellular drug-induced liver injury secondary to INH is evident with a variable latency period up to 6 months and is more frequent in alcoholics and patients taking certain other medications, such as barbiturates, rifampin, and pyrazinamide. If the clinical threshold of encephalopathy is reached, severe hepatic injury is likely to be fatal or to require liver transplantation. Liver biopsy reveals morphologic changes similar to those of viral hepatitis or bridging hepatic necrosis. Substantial liver injury appears to be age-related, increasing substantially after age 35; the highest frequency is in patients over age 50. Even for patients >50 years of age monitored carefully during therapy, hepatotoxicity occurs in only ~2%, well below the risk estimate derived from earlier experiences. Many public health programs that require INH prophylaxis for a positive tuberculin skin test or Quantiferon test include monthly monitoring of aminotransferase levels, although this practice has been called into question. Even more effective in limiting serious outcomes may be encouraging patients to be alert for symptoms such as nausea, fatigue, or jaundice, because most fatalities occur in the setting of continued INH use despite clinically apparent illness.
++
++
++
You are caring for a 48-year-old former drug user with chronic hepatitis C who is currently on no medications and whose most recent cardiovascular risk profile suggests he would benefit from initiation of statin therapy. His most recent liver function tests reveal high normal AST and ALT, normal alkaline phosphatase, and normal INR. He has not initiated antiviral therapy because of insurance issues with the new expensive curative therapies. Which of the following statements regarding statin therapy is true?
++
++
++
A. Monitoring of aminotransferase levels should be initiated in patients starting statin therapy.
++
++
B. Overall, between 5% and 10% of patients receiving statins develop mild reversible elevations in aminotransferase levels.
++
++
C. Statins are not contraindicated in patients with chronic hepatitis C.
++
++
D. Statins should be discontinued in asymptomatic patients who develop an isolated elevation in aminotransferase activity.
+
++
The answer is C. (Chap. 361) HMG-COA reductase inhibitors, or statins, may cause an idiosyncratic mixed hepatocellular and cholestatic reaction. Between 1% and 2% of patients taking lovastatin, simvastatin, pravastatin, fluvastatin, or one of the newer statin drugs for the treatment of hypercholesterolemia experience asymptomatic, reversible elevations (greater than threefold) of aminotransferase activity. Acute hepatitis-like histologic changes, centrilobular necrosis, and centrilobular cholestasis have been described in a very small number of cases. In a larger proportion, minor aminotransferase elevations appear during the first several weeks of therapy. Careful laboratory monitoring can distinguish between patients with minor, transitory changes, who may continue therapy, and those with more profound and sustained abnormalities, who should discontinue therapy. Because clinically meaningful aminotransferase elevations are so rare after statin use and do not differ in meta-analyses from the frequency of such laboratory abnormalities in placebo recipients, the National Lipid Association’s Safety Task Force concluded that liver test monitoring was not necessary in patients treated with statins and that statin therapy need not be discontinued in patients found to have asymptomatic isolated aminotransferase elevations during therapy. Statin hepatotoxicity is not increased in patients with chronic hepatitis C, hepatic steatosis, or other underlying liver diseases, and statins can be used safely in these patients.
++
++
++
All of the following are likely causes of chronic hepatitis EXCEPT:
++
++
++
++
++
++
++
++
++
++
++
+
++
The answer is B. (Chap. 362) Chronic hepatitis represents a series of liver disorders of varying causes and severity in which hepatic inflammation and necrosis continue for at least 6 months. Milder forms are nonprogressive or only slowly progressive, whereas more severe forms may be associated with scarring and architectural reorganization, which, when advanced, lead ultimately to cirrhosis. Several categories of chronic hepatitis have been recognized. These include chronic viral hepatitis, drug-induced chronic hepatitis, and autoimmune chronic hepatitis. In many cases, clinical and laboratory features are insufficient to allow assignment into one of these three categories; these “idiopathic” cases are also believed to represent autoimmune chronic hepatitis (Table VIII-62). Finally, clinical and laboratory features of chronic hepatitis are observed occasionally in patients with such hereditary/metabolic disorders as Wilson disease (copper overload), α1-antitrypsin deficiency (Chaps. 365 and 429), and nonalcoholic fatty liver disease (Chap. 367e), and even occasionally in patients with alcoholic liver injury (Chap. 363). Both of the enterically transmitted forms of viral hepatitis, hepatitis A and E, are self-limited and do not cause chronic hepatitis.
++
++
++
++
A 38-year-old woman is evaluated for elevated transaminase levels that were identified during routine laboratory testing for life insurance. She is originally from Thailand and immigrated to the United States 10 years previously. She has been married to an American for the past 12 years, meeting him while he was living abroad for business. She previously worked in Thailand as a deputy tourism minister for the government but is not currently employed. She has no significant past medical history. She had one uncomplicated pregnancy at the age of 22. When queried about risk factors for liver disease, she denies alcohol intake or drug abuse. She has never had a blood transfusion. She recalls an episode of jaundice that she did not seek evaluation for about 15 years ago. It resolved spontaneously. She currently feels well, and her husband wished to have her added to his life insurance policy. There is no stigmata of chronic liver disease. Her laboratory studies reveal an AST of 346 IU/L, ALT of 412 IU/L, alkaline phosphatase of 98 IU/L, and total bilirubin 1.5 of mg/dL. Further workup includes the following viral studies: hepatitis A IgG positive, hepatitis B surface antigen positive, hepatitis B e antigen positive, anti-hepatitis B virus core IgG positive, and hepatitis C IgG negative. The HBV DNA level is 4.8 × 104 IU/mL. Which of the following medications is indicated for this patient?
++
++
++
++
++
++
++
++
++
++
++
E. No treatment is necessary
+
++
The answer is B. (Chap. 362) The patient in this scenario has evidence of chronic active hepatitis B virus (HBV) infection. The presence of hepatitis B e antigen (HBeAg) is indicative of ongoing viral replication, and individuals with HBeAg positivity typically have high levels of HBV DNA on testing. The spectrum of clinical infection in chronic hepatitis B is quite variable, and often, individuals are asymptomatic with elevated liver enzymes identified on testing for other reasons. Thus, the decision to treat chronic HBV infection should not be based on clinical features. Most experts recommend treatment of HBeAg-positive chronic HBV infection with HBV DNA levels >2 × 104 IU/mL if the ALT is elevated greater than twice the upper limit of normal (Table VIII-63). To date, seven drugs have been approved for treatment of chronic HBV: injectable interferon (IFN) α; pegylated IFN (long-acting IFN bound to polyethylene glycol [PEG IFN]); and the oral agents lamivudine, adefovir dipivoxil, entecavir, telbivudine, and tenofovir. PEG IFN, entecavir, or tenofovir is recommended as first-line therapy. Simeprevir is a protease inhibitor that is approved for the treatment of hepatitis C virus infection. Acyclovir is used to treat herpes simplex viral infections, and ritonavir is a protease inhibitor used to treat HIV infection. The patient’s husband should also be screened for HBV given the continued viremia.
++
++
++
++
A 46-year-old man is known to have chronic hepatitis C virus (HCV) infection. He is a former intravenous drug user for more than 20 years who has been abstinent from drug use for 1 year. He was treated for tricuspid valve endocarditis 3 years previously. He does not know when he acquired HCV. His laboratory studies show a positive HCV IgG antibody with a viral load of greater than 1 million copies. The virus is genotype 2. His AST is 82 IU/L, and his ALT is 74 IU/L. He undergoes liver biopsy, which demonstrates a moderate degree of bridging fibrosis. Which of the following is the most predictive of the development of cirrhosis?
++
++
++
A. Abnormal transaminases
++
++
B. Bridging fibrosis on liver biopsy
++
++
++
++
D. History of bacterial endocarditis
++
++
E. History of intravenous drug use
+
++
The answer is B. (Chap. 362) Chronic hepatitis develops in about 85% of all individuals affected with hepatitis C virus (HCV), and 20%–25% of these individuals will progress to cirrhosis over about 20 years. Among those infected with HCV, about one-third of individuals will have normal or near-normal levels of aminotransferases, although liver biopsy demonstrates active hepatitis in an many as one-half of patients. Moreover, about 25% of individuals with normal aminotransferase levels at one point in time will develop elevations in these enzymes later and can develop progressive liver disease. Thus, normal aminotransferase levels at a single point in time do not definitively rule out the possibility that cirrhosis can develop. Progression to end-stage liver disease in individuals with chronic HCV hepatitis is more likely in older individuals and in patients with longer duration of infection, advanced histologic stage and grade, genotype 1 infection, more complex quasi-species diversity, concomitant other liver disease, HIV infection, and obesity. Among these factors, the best prognostic indicator for the development of progressive liver disease is liver histology. Specifically, patients who have moderate to severe inflammation or necrosis, including septal or bridging fibrosis, have the greatest risk of developing cirrhosis over the course of 10–20 years.
++
++
++
A 34-year-old woman is evaluated for fatigue, malaise, arthralgias, and a 10-lb weight loss over the past 6–8 weeks. She has no past medical history. Since feeling poorly, she has taken approximately one or two tablets of acetaminophen 500 mg daily. On physical examination, her temperature is 100.2°F, respiratory rate is 18 breaths/min, blood pressure is 100/48 mmHg, heart rate is 92 bpm, and oxygen saturation is 96% on room air. She has scleral icterus. Her liver edge is palpable 3 cm below the right costal margin. It is smooth and tender. The spleen is not enlarged. She has mild synovitis in the small joints of her hands. Her AST is 542 IU/L, ALT is 657 IU/L, alkaline phosphatase is 102 IU/L, total bilirubin is 5.3 mg/dL, and direct bilirubin is 4.8 mg/dL. Which of the following tests would be LEAST likely to be positive in this diagnosis?
++
++
++
A. Antinuclear antibodies in a homogeneous pattern
++
++
B. Anti-liver/kidney microsomal antibodies
++
++
C. Antimitochondrial antibodies
++
++
D. Hypergammaglobulinemia
++
++
+
++
The answer is C. (Chap. 362) Three types of autoimmune hepatitis have been identified based on clinical and laboratory characteristics. Type I autoimmune hepatitis is typically a disorder seen in young women. The clinical characteristics can be variable from those of chronic hepatitis to fulminant hepatic failure, and many of the features are difficult to distinguish from other causes of chronic hepatitis. In some individuals, extrahepatic manifestations, including fatigue, malaise, weight loss, anorexia, and arthralgias, can be quite prominent. Liver enzymes are elevated but may not correlate with the clinical severity of disease. In more severe cases, elevations in serum bilirubin between 3 and 10 mg/dL can be seen. Hypoalbuminemia occurs in advanced disease, and hypergammaglobulinemia (>2.5 g/dL) is common. The circulating antibody profile in autoimmune hepatitis depends to some extent on the type of hepatitis. Antinuclear antibodies are positive in a homogeneous staining pattern almost invariably in the disease, and rheumatoid factor is also common. Perinuclear antineutrophilic cytoplasmic antibody may be positive, but in an atypical fashion. Anti–smooth muscle antibodies and anti-liver/kidney microsomal antibodies are frequently seen, but these are nonspecific because other causes of chronic hepatitis can lead to positivity of these enzymes. Because of the lack of a specific autoimmune profile, the diagnostic criteria for autoimmune hepatitis incorporate a variety of clinical and laboratory features. Specific features that argue against this diagnosis include prominent alkaline phosphatase elevation, presence of mitochondrial antibodies, markers of viral hepatitis, history of hepatotoxic drugs or excess alcohol intake, histologic evidence of bile duct injury, or atypical biopsy features including excess hepatic iron, fatty infiltration, and viral inclusions. Antimitochondrial antibodies are typically seen in primary biliary cirrhosis.
++
++
++
In chronic hepatitis B virus infection, presence of hepatitis B e antigen signifies which of the following?
++
++
++
A. Development of liver fibrosis leading to cirrhosis
++
++
B. Dominant viral population is less virulent and less transmissible
++
++
C. Increased likelihood of an acute flare in the next 1–2 weeks
++
++
D. Ongoing viral replication
++
++
+
++
The answer is D. (Chaps. 360 and 362) In the course of acute hepatitis B, HBeAg positivity is common and usually transient. Persistence of HBeAg in the serum for >3 months indicates an increased likelihood of development of chronic hepatitis B. In chronic hepatitis B, presence of HBeAg in the serum indicates ongoing viral replication and increased infectivity. It is also a surrogate for inflammatory liver injury but not fibrosis. The development of antibody to HBeAg (anti-HBe) is indicative of the nonreplicative phase of HBV infection. During this phase, intact virions do not circulate and infectivity is less. Currently, quantification of HBV DNA with polymerase chain reaction allows risk stratification because <103 virions/µL is the approximate threshold for liver injury and infectivity.
++
++
++
All of the following statements regarding alcoholic liver disease are true EXCEPT:
++
++
++
A. Fatty liver is present in >90% of daily and binge drinkers.
++
++
B. Hepatitis C infection worsens the prognosis of alcoholic liver disease.
++
++
C. Over 50% of alcoholics will develop alcoholic hepatitis.
++
++
D. Quantity and duration of alcohol consumption are the most important risk factors for the development of alcoholic liver disease.
++
++
E. The pathologic hallmarks of alcoholic liver disease are fatty liver, hepatitis, and cirrhosis.
+
++
The answer is C. (Chap. 363) The pathology of alcoholic liver disease consists of three major lesions, with the progressive injury rarely existing in a pure form: (1) fatty liver, (2) alcoholic hepatitis, and (3) cirrhosis. Fatty liver is present in >90% of daily as well as binge drinkers. A much smaller percentage of heavy drinkers will progress to alcoholic hepatitis, thought to be a precursor to cirrhosis. The prognosis of severe alcoholic liver disease is dismal; the mortality of patients with alcoholic hepatitis concurrent with cirrhosis is nearly 60% at 4 years. Although alcohol is considered a direct hepatotoxin, only between 10% and 20% of alcoholics will develop alcoholic hepatitis. The explanation for this apparent paradox is unclear but involves the complex interaction of facilitating factors, such as drinking patterns, diet, obesity, and gender. There are no diagnostic tools that can predict individual susceptibility to alcoholic liver disease. Quantity and duration of alcohol intake are the most important risk factors involved in the development of alcoholic liver disease. The roles of beverage type(s) (i.e., wine, beer, or spirits) and pattern of drinking (daily versus binge drinking) are less clear. Chronic infection with hepatitis C virus (HCV) is an important comorbidity in the progression of alcoholic liver disease to cirrhosis. Even moderate alcohol intake of 20–50 g/d increases the risk of cirrhosis and hepatocellular cancer in HCV-infected individuals. Patients with both alcoholic liver injury and HCV infection develop decompensated liver disease at a younger age and have poorer overall survival.
++
++
++
++
A 32-year-old woman is admitted to the hospital with fever, abdominal pain, and jaundice. She drinks approximately 6 beers daily and has recently increased her alcohol intake to more than 12 beers daily. She has no other substance abuse history and has no prior history of alcoholic liver disease or pancreatitis. She is not taking any medications. On physical examination, she appears ill and disheveled with a fruity odor to her breath. Her vital signs are: heart rate 122 bpm, blood pressure 95/56 mmHg, respiratory rate 22 breaths/min, temperature 101.2°F, and oxygen saturation 98% on room air. She has scleral icterus, and spider angiomata are present on the trunk. The liver edge is palpable 10 cm below the right costal margin. The liver is smooth and tender to palpation. The spleen is not palpable. No ascites or lower extremity edema is present. Laboratory studies demonstrate as AST of 431 IU/L, ALT of 198 IU/L, bilirubin of 8.6 mg/dL, alkaline phosphatase of 201 IU/L, amylase of 88 U/L, and lipase of 50 U/L. Total protein is 6.2 g/dL, and albumin is 2.8 g/dL. The prothrombin time is 29 seconds (control, 13 seconds) with INR of 2.2. What is the best approach to treatment of this patient?
++
++
++
A. Administer intravenous fluids, thiamine, and folate and observe for improvement in laboratory tests and clinical condition.
++
++
B. Administer intravenous fluids, thiamine, folate, and imipenem while awaiting blood culture results.
++
++
C. Administer prednisone 40 mg daily for 4 weeks before beginning a taper.
++
++
D. Consult surgery for management of acute cholecystitis.
++
++
E. Perform an abdominal CT with intravenous contrast to assess for necrotizing pancreatitis.
+
++
The answer is C. (Chap. 363) This patient presents with severe acute alcoholic hepatitis. In its earliest form, alcoholic liver disease is marked by fatty infiltration of the liver. In more acute alcoholic hepatitis, there is hepatocyte injury with balloon degeneration and necrosis. Many cases of alcoholic hepatitis are asymptomatic. However, as in this case, the severe manifestations can include fever, jaundice, spider nevi, and abdominal pain that can mimic an acute abdomen in its severity. On laboratory examination, the AST is typically elevated more than the ALT, although the total transaminase levels are rarely greater than 400 IU/L. Hyperbilirubinemia can be quite marked with lesser elevation in alkaline phosphatase. Hypoalbuminemia and coagulopathy are poor prognostic indicators. A discriminate function (DF) can be calculated as follows: (4.6 × the prolongation of prothrombin time above control) + serum bilirubin. A DF >32 is associated with poor prognosis and is an indication for treatment of acute alcoholic hepatitis. The Model for End-Stage Liver Disease (MELD) score can also be used for prognostication in acute alcoholic hepatitis, with a score greater than 21 being an indication for treatment as well. This patient has a DF of >80, indicating very severe disease and a poor prognosis. Complete abstinence from alcohol is imperative. Treatment with prednisone 40 mg daily (or prednisolone 32 mg daily) for 4 weeks should be initiated. Following the initial period, a taper should be achieved over a period of 4 weeks. The role of tumor necrosis factor (TNF)-α expression and receptor activity in alcoholic liver injury has led to an examination of TNF inhibition as an alternative to glucocorticoids for severe alcoholic hepatitis. The nonspecific TNF inhibitor pentoxifylline (400 mg three times daily for 4 weeks) demonstrated improved survival in the therapy of severe alcoholic hepatitis, primarily due to a decrease in hepatorenal syndrome. Monoclonal antibodies that neutralize serum TNF-α should not be used in alcoholic hepatitis because of studies reporting increased deaths secondary to infection and renal failure. Liver transplantation is an accepted indication for treatment in selected and motivated patients with end-stage cirrhosis. Outcomes are equal or superior to other indications for transplantation.
++
++
++
Which of the following statements regarding nonalcoholic fatty liver disease (NAFLD) is true?
++
++
++
A. Imaging studies suggest that fatty liver is present to some degree in 10% of adult Americans.
++
++
B. NAFLD does not occur in lean individuals.
++
++
C. NAFLD is more common in African Americans than Hispanic Americans.
++
++
D. NAFLD is strongly associated with obesity and insulin resistance.
++
++
E. While common in the United States, NAFLD is uncommon in other countries.
+
++
The answer is D. (Chap. 364) Nonalcoholic fatty liver disease (NAFLD) is the most common chronic liver disease in many parts of the world, including the United States. Population-based abdominal imaging studies have demonstrated fatty liver in at least 25% of American adults. Because the vast majority of these subjects deny hazardous levels of alcohol consumption (defined as greater than one drink per day in women or two drinks per day in men), they are considered to have NAFLD. NAFLD is strongly associated with overweight/obesity and insulin resistance. However, it can also occur in lean individuals and is particularly common in those with a paucity of adipose depots (i.e., lipodystrophy). Ethnic/racial factors also appear to influence liver fat accumulation; the documented prevalence of NAFLD is lowest in African Americans (~25%), highest in Americans of Hispanic ancestry (~50%), and intermediate in American whites (~33%).
++
++
++
A 44-year-old man seeks evaluation for an abnormal finding on ultrasonography. He has a history of type 2 diabetes mellitus and is on insulin therapy. Last week, he was evaluated in the emergency department for mid-epigastric pain likely due to NSAID therapy for muscle aches (he recently started exercising because his wife told him to lose weight). During the evaluation, an abdominal ultrasound showed marked fatty infiltration of the liver. Laboratory studies show his transaminases are 2× normal with normal alkaline phosphatase, bilirubin, and prothrombin time. Other than insulin, he takes no medications, does not consume alcohol or illicit drugs, and has no family history of liver disease. On physical examination, he is obese (BMI, 32 kg/m2) with normal vital signs and no other abnormalities. You think he likely has NAFLD. All of the following statements regarding his potential therapy are true EXCEPT:
++
++
++
A. Bariatric surgery is safe in patients with NAFLD.
++
++
B. Exercise may reduce hepatic steatosis.
++
++
C. Statins may worsen inflammation in NAFLD.
++
++
D. There are no Food and Drug Administration–approved therapies for NAFLD.
++
++
E. Vitamin E may reduce aminotransferase levels and hepatic steatosis.
+
++
The answer is C. (Chap. 364) At present, there are no Food and Drug Administration–approved therapies for the treatment of NAFLD. Thus, the current approach to NAFLD management focuses on treatment to improve the risk factors for nonalcoholic steatohepatitis (NASH; i.e., obesity, insulin resistance, metabolic syndrome, dyslipidemia). Based on our understanding of the natural history of NAFLD, only patients with NASH or those with features of hepatic fibrosis on liver biopsy are considered currently for targeted pharmacologic therapies. Lifestyle changes and dietary modification are the foundation for NAFLD treatment. Many studies indicate that lifestyle modification can improve serum aminotransferases and hepatic steatosis, with loss of at least 3%–5% of body weight improving steatosis, but greater weight loss (up to 10%) necessary to improve steatohepatitis. The benefits of different dietary macronutrient contents (e.g., low-carbohydrate vs. low-fat diets, saturated vs. unsaturated fat diets) and different intensities of calorie restriction appear to be comparable. In adults with NAFLD, exercise regimens that improve fitness may be sufficient to reduce hepatic steatosis, but their impact on other aspects of liver histology remains unknown. Antioxidants have also been evaluated for the treatment of NAFLD because oxidant stress is thought to contribute to the pathogenesis of NASH. Vitamin E, an inexpensive yet potent antioxidant, has been examined in several small pediatric and adult studies with varying results. In all of those studies, vitamin E was well tolerated, and most showed modest improvements in aminotransferase levels, radiographic features of hepatic steatosis, and/or histologic features of NASH. Statins are an important class of agents to treat dyslipidemia and decrease cardiovascular risk. There is no evidence to suggest that statins cause liver failure in patients with any chronic liver disease, including NAFLD. The incidence of liver enzyme elevations in NAFLD patients taking statins is also no different than that of healthy controls or patients with other chronic liver diseases. Moreover, several studies have suggested that statins may improve aminotransferases and histology in patients with NASH. However, there is continued reluctance to use statins in patients with NAFLD. The lack of evidence that statins harm the liver in NAFLD patients, combined with the increased risk for cardiovascular morbidity and mortality in NAFLD patients, warrants the use of statins to treat dyslipidemia in patients with NAFLD/NASH. Although interest in bariatric surgery as a treatment for NAFLD exists, a recently published Cochrane review concluded that lack of randomized clinical trials or adequate clinical studies prevents definitive assessment of benefits and harms of bariatric surgery as a treatment for NASH. Most studies of bariatric surgery have shown that it is generally safe in individuals with well-compensated chronic liver disease and improves hepatic steatosis and necroinflammation (i.e., features of NAFLD/NASH); however, effects on hepatic fibrosis have been variable.
++
++
++
All of the following statements regarding alcohol-induced liver disease are true EXCEPT:
++
++
++
A. Alcohol-induced cirrhosis is characterized by predominantly large (>2 cm) nodules.
++
++
B. Alcohol use is the most common cause of cirrhosis in the United States.
++
++
C. Chronic alcohol use can cause liver fibrosis in the absence of accompanying inflammation.
++
++
D. Excessive alcohol use can cause acute hepatitis.
++
++
E. Excessive alcohol use can worsen liver disease due to hemochromatosis.
+
++
The answer is A. (Chap. 365) Alcohol is the most commonly used drug in the United States, and more than two-thirds of adults drink alcohol each year. Thirty percent have had a binge within the past month, and over 7% of adults regularly consume more than two drinks per day. Unfortunately, more than 14 million adults in the United States meet the diagnostic criteria for alcohol abuse or dependence. In the United States, chronic liver disease is the 10th most common cause of death in adults, and alcoholic cirrhosis accounts for approximately 40% of deaths due to cirrhosis. Excessive chronic alcohol use can cause several different types of chronic liver disease, including alcoholic fatty liver, alcoholic hepatitis, and alcoholic cirrhosis. Furthermore, use of excessive alcohol can contribute to liver damage in patients with other liver diseases, such as hepatitis C, hemochromatosis, and fatty liver disease related to obesity. Chronic alcohol use can produce fibrosis in the absence of accompanying inflammation and/or necrosis. Fibrosis can be centrilobular, pericellular, or periportal. When fibrosis reaches a certain degree, there is disruption of the normal liver architecture and replacement of liver cells by regenerative nodules. In alcoholic cirrhosis, the nodules are usually <3 mm in diameter; this form of cirrhosis is referred to as micronodular. With cessation of alcohol use, larger nodules may form, resulting in a mixed micronodular and macronodular cirrhosis.
++
++
++
A 64-year-old man is admitted to the ICU with a large GI bleed. EGD reveals esophageal varices. He is confused and unable to provide any history. His physical examination is notable for abnormal hand findings as shown in Figures VIII-72A and VIII-72B. Ultrasound demonstrates a small liver consistent with cirrhosis. Which of the following is the most likely cause of his cirrhosis?
+
++
++
++
+
++
++
++
++
++
++
++
C. Chronic hepatitis C infection
++
++
++
++
E. Primary biliary cirrhosis
+
++
The answer is A. (Chap. 365) The patient’s hands show palmar erythema typical of alcoholic cirrhosis. This finding and the gynecomastia and testicular atrophy make alcoholic cirrhosis the most likely diagnosis. The testicular atrophy may be due to either a direct toxic effect of alcohol or hormonal effects. All of the other items may also cause liver cirrhosis (Table VIII-72).
++
++
++
++
A 64-year-old man is admitted to the ICU with a large GI bleed. EGD reveals esophageal varices. He is confused and unable to provide any history. His physical examination is notable for abnormal hand findings as shown in Figures VIII-72A and VIII-72B. Ultrasound demonstrates a small liver consistent with cirrhosis. In the patient described all of the following may be used to control his variceal bleeding EXCEPT:
+
++
++
++
+
++
++
++
A. Endoscopic sclerotherapy
++
++
B. Endoscopic variceal ligation
++
++
++
++
++
++
E. Transjugular intrahepatic portosystemic shunt
+
++
The answer is D. (Chap. 365) The approach to patients once they have had a variceal bleed is first to treat the acute bleed, which can be life-threatening, and then to prevent further bleeding. The medical management of acute variceal hemorrhage includes the use of vasoconstricting agents, usually somatostatin or octreotide. Balloon tamponade (Sengstaken-Blakemore tube or Minnesota tube) can be used in patients who cannot receive endoscopic therapy immediately or who need stabilization prior to endoscopic therapy. Endoscopic intervention is used as first-line treatment to control bleeding acutely. Some endoscopists will use variceal injection therapy (sclerotherapy) as initial therapy, particularly when bleeding is vigorous. Variceal band ligation is used to control acute bleeding in over 90% of cases and should be repeated until obliteration of all varices is accomplished. When esophageal varices extend into the proximal stomach, band ligation is less successful. In these situations, when bleeding continues from gastric varices, consideration for a transjugular intrahepatic portosystemic shunt (TIPS) should be made. This offers an alternative to surgery for acute decompression of portal hypertension. Encephalopathy can occur in as many as 20% of patients after TIPS and is particularly problematic in elderly patients and in patients with preexisting encephalopathy. TIPS should be reserved for individuals who fail endoscopic or medical management or who are poor surgical risks. TIPS can sometimes be used as a bridge to transplantation. Prevention of further bleeding is usually accomplished with repeated variceal band ligation until varices are obliterated. B-Blockers, such as propranolol and nadolol, have been shown to decrease the risk of recurrent variceal bleeding and reduce mortality from a subsequent bleed but should not be used in the setting of the acutely bleeding patient.
++
++
++
You are following a 44-year-old woman with cirrhosis due to chronic hepatitis C infection. To date, she has no evidence of portal hypertension, and her disease is well controlled on her current medical regimen. All of the following new findings on a diagnostic study are suggestive of the development of portal hypertension EXCEPT:
++
++
++
A. 15-cmHg gradient between wedged and free hepatic vein pressures
++
++
++
++
C. Enlarged spleen on physical examination
++
++
D. Left atrial dilation on echocardiogram
++
++
+
++
The answer is D. (Chap. 365) In patients with cirrhosis who are being followed chronically, the development of portal hypertension is usually revealed by the presence of thrombocytopenia; the appearance of an enlarged spleen; or the development of ascites, encephalopathy, and/or esophageal varices with or without bleeding. In previously undiagnosed patients, any of these features should prompt further evaluation to determine the presence of portal hypertension and liver disease. Varices should be identified by endoscopy. Abdominal imaging, either by CT or magnetic resonance imaging (MRI), can be helpful in demonstrating a nodular liver and in finding changes of portal hypertension with intra-abdominal collateral circulation. If necessary, interventional radiologic procedures can be performed to determine wedged and free hepatic vein pressures that will allow for the calculation of a wedged-to-free gradient, which is equivalent to the portal pressure. The average normal wedged-to-free gradient is 5 mmHg, and patients with a gradient >12 mmHg are at risk for variceal hemorrhage. While dilation of the right atrium may be found in cases of cardiac cirrhosis, dilation of the left atrium is characteristic of left ventricular failure.
++
++
++
A 63-year-old man with cirrhosis and portal hypertension due to hemochromatosis presents with altered mental status. He has chronic ascites controlled with diet and spironolactone. He has a history of one esophageal bleed but none since starting propranolol. His family reports that over the last 2 days, he has become more confused, but he has had no melena or hematemesis. He is afebrile with normal vital signs, and physical examination is notable for ascites, asterixis, and being oriented only to person. His laboratory examination is notable for a hemoglobin of 10.1 (baseline, 9.5), creatinine of 1.4 (baseline, 1.4), and blood urea nitrogen of 45 (baseline, 18). A paracentesis is performed that yields reveals clear fluid with 800 WBC (40% neutrophils). Which of the following is the most indicated therapy?
++
++
++
++
++
++
++
++
++
++
++
+
++
The answer is B. (Chap. 365) Spontaneous bacterial peritonitis (SBP) is a common and severe complication of ascites characterized by spontaneous infection of the ascitic fluid without an intra-abdominal source. In patients with cirrhosis and ascites severe enough for hospitalization, SBP can occur in up to 30% of individuals and can have a 25% in-hospital mortality rate. Bacterial translocation is the presumed mechanism for development of SBP, with gut flora traversing the intestine into mesenteric lymph nodes, leading to bacteremia and seeding of the ascitic fluid. The most common organisms are E coli and other gut bacteria; however, gram-positive bacteria, including Streptococcus viridans, Staphylococcus aureus, and Enterococcus spp., can also be found. If more than two organisms are identified, secondary bacterial peritonitis due to a perforated viscus should be considered. The diagnosis of SBP is made when the fluid sample has an absolute neutrophil count >250/μL. In this case, the patient has an absolute neutrophil count of 320/μL (800 × 0.4). Patients with ascites may present with fever, altered mental status, elevated white blood cell count, and abdominal pain or discomfort, or they may present without any of these features. Therefore, it is necessary to have a high degree of clinical suspicion, and peritoneal taps are important for making the diagnosis. Treatment is with a second-generation cephalosporin, with cefotaxime being the most commonly used antibiotic. In patients with variceal hemorrhage, the frequency of SBP is significantly increased, and prophylaxis against SBP is recommended when a patient presents with upper GI bleeding. Furthermore, in patients who have had an episode(s) of SBP and recovered, once-weekly administration of antibiotics is used as prophylaxis for recurrent SBP. There is no indication for hemodialysis with the normal serum creatine or EGD with no history of bleeding and a stable hemoglobin. Blood urea nitrogen (BUN) may increase as a result of the infection. Similarly, although the BUN is elevated and the patient has altered mental status, lactulose would not treat the primary disorder causing the altered mental status. Given the likely diagnosis of SBP, empiric therapy for meningitis is not warranted at this time.
++
++
++
A 48-year-old woman presents complaining of fatigue and itching. She has been tired for the past 6 months and recently has developed itching diffusely. It is worse in the evening hours but is intermittent. She does not note it to be worse following hot baths or showers. Her past medical history is significant only for hypothyroidism for which she takes levothyroxine 125 μg daily. On physical examination, she has mild jaundice and scleral icterus. The liver is enlarged to 15 cm on palpation and is palpable 5 cm below the right costal margin. Xanthomas are seen on both elbows. Hyperpigmentation is noticeable on the trunk and arms where the patient has excoriations. Laboratory studies demonstrate the following: WBC 8,900/µL, hemoglobin 13.3 g/dL, hematocrit 41.6%, and platelets 160,000/µL. The creatinine is 1.2 mg/dL. The AST is 52 IU/L, ALT is 62 IU/L, alkaline phosphatase is 216 IU/L, total bilirubin is 3.2 mg/dL, and direct bilirubin is 2.9 mg/dL. The total protein is 8.2 g/dL, and albumin is 3.9 U/L. The thyroid-stimulating hormone is 4.5 U/mL. Antimitochondrial antibodies are positive. Perinuclear antineutrophil cytoplasmic antibodies (ANCA) and cytoplasmic ANCA are negative. What is the most likely cause of the patient’s symptoms?
++
++
++
++
++
++
++
C. Primary biliary cirrhosis
++
++
D. Primary sclerosis cholangitis
++
++
E. Uncontrolled hypothyroidism
+
++
The answer is C. (Chap. 365) The clinical presentation is consistent with a cholestatic picture, which can present with painless jaundice and pruritus. The pruritus can be prominent and is present in 50% of individuals at the time of diagnosis. The pruritus is typically intermittent and worse in the evening. There is no other prominent association, such as following hot baths or showers, which occurs in polycythemia vera. Other causes of pruritus outside of cholestasis include lymphoma and uncontrolled hypo- or hyperthyroidism. However, the laboratory studies in this patient clearly represent cholestasis with an elevation in alkaline phosphatase and bilirubin. The clinical characteristics are more commonly seen in primary biliary cirrhosis compared to primary sclerosis cholangitis as the patient is a middle-aged female with positive antimitochondrial antibodies. In contrast, primary sclerosing cholangitis is associated with positive perinuclear antineutrophil cytoplasmic antibodies in 65% of patients, and 50% of individuals with primary sclerosing cholangitis have a history of ulcerative colitis.
++
++
++
A 42-year-old man with cirrhosis related to hepatitis C and alcohol abuse has ascites requiring frequent large-volume paracentesis. All of the following therapies would be indicated for this patient EXCEPT:
++
++
++
A. Fluid restriction to less than 2 L daily
++
++
B. Furosemide 40 mg daily
++
++
C. Sodium restriction to less than 2 g daily
++
++
D. Spironolactone 100 mg daily
++
++
E. Transjugular intrahepatic portosystemic shunt if medical therapy fails
+
++
The answer is A. (Chap. 365) The cornerstone of the management of ascites is sodium restriction to less than 2 g daily. A common misconception is to institute a fluid restriction as well. However, this is neither effective nor necessary. With a sodium restriction to 2 g daily, most mild ascites can be managed quite well. If sodium restriction alone fails to correct ascites, then initiation of diuretics is required. Spironolactone at a dose of 100–200 mg daily is the initial diuretic used for ascites and can be titrated as high as 400–600 mg daily if tolerated. Loop diuretics can be added to spironolactone. The typical agent is furosemide beginning at 40–80 mg daily with maximum doses of about 120–160 mg daily. Care must be taken to avoid renal dysfunction with loop diuretics, and higher doses may not be tolerated. If ascites is refractory to these treatments, TIPS can be considered. This procedure creates a direct portocaval shunt by introducing an expandable metal stent from the hepatic veins through the substance of the liver into the portal veins. Thus, TIPS decreases portal pressures, which in turn decrease ascites and the risk of variceal bleeding. However, hepatic encephalopathy typically worsens following TIPS.
++
++
++
Which of the following statements about cardiac cirrhosis is true?
++
++
++
A. AST and ALT levels may mimic the very high levels seen in acute viral hepatitis.
++
++
B. Budd-Chiari syndrome cannot be distinguished clinically from cardiac cirrhosis.
++
++
C. Echocardiography is the gold standard for diagnosing constrictive pericarditis as a cause of cirrhosis.
++
++
D. Prolonged passive congestion from right-sided heart failure results in congestion and necrosis of portal triads, resulting in subsequent fibrosis.
++
++
E. Veno-occlusive disease can be confused with cardiac cirrhosis and is a major cause of morbidity and mortality in patients undergoing liver transplantation.
+
++
The answer is A. (Chap. 365) Severe right-sided heart failure may lead to chronic liver injury and cardiac cirrhosis. Elevated venous pressure leads to congestion of the hepatic sinusoids and of the central vein and centrilobular hepatocytes. Centrilobular fibrosis develops, and fibrosis extends outward from the central vein, not the portal triads. Gross examination of the liver shows a pattern of “nutmeg liver.” Although transaminases are typically mildly elevated, severe congestion, particularly associated with hypotension, may result in dramatic elevation of AST and ALT 50- to 100-fold above normal. Budd-Chiari syndrome, or occlusion of the hepatic veins or inferior vena cava, may be confused with congestive hepatopathy. However, the signs and symptoms of congestive heart failure are absent in patients with Budd-Chiari syndrome, and these patients can be easily distinguished clinically from those with heart failure. Veno-occlusive disease may result from hepatic irradiation and high-dose chemotherapy in preparation for hematopoietic stem cell transplantation. It is not a typical complication of liver transplantation. Although echocardiography is a useful tool for assessing left and right ventricular function, findings may be unimpressive in patients with constrictive pericarditis. A high index of suspicion for constrictive pericarditis (e.g., prior episodes of pericarditis, mediastinal irradiation) should lead to a right-sided heart catheterization with demonstration of the “square root sign,” limitation of right heart filling pressure in diastole that is suggestive of restrictive cardiomyopathy. Cardiac MRI may also be helpful in determining which patients should proceed to cardiac surgery.
++
++
++
You are asked to consult on a 62-year-old white woman with pruritus for 4 months. She has noted progressive fatigue and a 5-lb weight loss. She has intermittent nausea but no vomiting and denies changes in her bowel habits. There is no history of prior alcohol use, blood transfusions, or illicit drug use. The patient is widowed and had two heterosexual partners in her lifetime. Her past medical history is significant only for hypothyroidism, for which she takes levothyroxine. Her family history is unremarkable. On examination, she is mildly icteric. She has spider angiomata on her torso. You palpate a nodular liver edge 2 cm below the right costal margin. The remainder of the examination is unremarkable. A right upper quadrant ultrasound confirms your suspicion of cirrhosis. You order a complete blood count and a comprehensive metabolic panel. What is the most appropriate next test?
++
++
++
++
++
B. Antimitochondrial antibodies
++
++
C. Endoscopic retrograde cholangiopancreatography
++
++
D. Hepatitis B serologies
++
++
+
++
The answer is B. (Chap. 365) The presence of cirrhosis in an elderly woman with no prior risk factors for viral or alcoholic cirrhosis should raise the possibility of primary biliary cirrhosis (PBC). PBC is characterized by chronic inflammation and fibrous obliteration of intrahepatic ductules. The cause is unknown, but autoimmunity is assumed because there is an association with other autoimmune disorders, such as autoimmune thyroiditis, CREST syndrome, and sicca syndrome. The vast majority of patients with symptomatic disease are women. The antimitochondrial antibody (AMA) test is positive in over 90% of patients with PBC and only rarely is positive in other conditions. This makes it the most useful initial test in the diagnosis of PBC. Because there are false-positive results, if AMA is positive, a liver biopsy is performed to confirm the diagnosis. The 24-hour urine copper collection is useful in the diagnosis of Wilson disease. Hepatic failure from Wilson disease typically occurs before age 50 years. Hemochromatosis may result in cirrhosis. It is associated with lethargy, fatigue, loss of libido, discoloration of the skin, arthralgias, diabetes, and cardiomyopathy. Ferritin levels are usually increased, and the most suggestive laboratory abnormality is an elevated transferrin saturation percentage. Although hemochromatosis is a possible diagnosis in this case, PBC is more likely in light of the clinical scenario. Although chronic hepatitis B and hepatitis C are certainly in the differential diagnosis and must be ruled out, they are unlikely because of the patient’s history and lack of risk factors.
++
++
++
All of the following are potential indications for liver transplantation EXCEPT:
++
++
++
++
++
++
++
C. Primary biliary cirrhosis
++
++
D. Primary hepatocellular carcinoma
++
++
E. Primary sclerosing cholangitis
+
++
The answer is B. (Chap. 368) Liver transplantation is indicated in adults for end-stage cirrhosis of all causes. Routine candidates for liver transplantation are patients with alcoholic cirrhosis, chronic viral hepatitis, and primary hepatocellular malignancies. Although all three of these categories are considered to be high risk, liver transplantation can be offered to carefully selected patients. Currently, chronic hepatitis C and alcoholic liver disease are the most common indications for liver transplantation, accounting for over 40% of all adult candidates who undergo the procedure. Patients with alcoholic cirrhosis can be considered as candidates for transplantation if they meet strict criteria for abstinence and reform; however, these criteria still do not prevent recidivism in up to a quarter of cases. In sclerosing cholangitis and Caroli disease (multiple cystic dilatations of the intrahepatic biliary tree), recurrent infections and sepsis associated with inflammatory and fibrotic obstruction of the biliary tree may be an indication for transplantation. Because prior biliary surgery complicates and is a relative contraindication for liver transplantation, surgical diversion of the biliary tree has been all but abandoned for patients with sclerosing cholangitis. Patients with chronic hepatitis C have early allograft and patient survival comparable to those of other subsets of patients after transplantation; however, reinfection in the donor organ is universal, recurrent hepatitis C is insidiously progressive, allograft cirrhosis develops in 20%–30% at 5 years, and cirrhosis and late organ failure occur at a higher frequency beyond 5 years. With the introduction of highly effective direct-acting antiviral agents targeting hepatitis C, it is expected that allograft outcomes will improve significantly in the coming years. In patients with chronic hepatitis B, in the absence of measures to prevent recurrent hepatitis B, survival after transplantation is reduced by approximately 10%–20%; however, prophylactic use of hepatitis B immune globulin (HBIg) during and after transplantation increases the success of transplantation to a level comparable to that seen in patients with nonviral causes of liver decompensation. Specific oral antiviral drugs can be used both for prophylaxis against and for treatment of recurrent hepatitis B. Patients with primary hepatocellular carcinoma with a single tumor <5 cm or three or fewer tumors of <3 cm have 5-year recurrence-free survival rates similar to those with nonmalignant disease. Because of the high rate of recurrent disease after transplantation, patients with cholangiocarcinoma are not transplantation candidates.
++
++
++
Which of the following patients is highest priority for liver transplantation?
++
++
++
A. A 24-year-old woman with cirrhosis due to autoimmune hepatitis. She has been on the transplant list for 2 months and now has an elevated bilirubin, INR, and creatinine.
++
++
B. A 38-year-old woman with chronic hepatitis C and normal bilirubin and INR.
++
++
C. A 49-year-old man with alcoholic cirrhosis who has been on the transplant list for 6 months. He has had two esophageal variceal bleeds.
++
++
D. A 59-year-old man with a history of hyperlipidemia who was admitted to the ICU 2 days ago with fulminant hepatic failure due to mistakenly ingesting Amanita mushrooms from his lawn.
++
++
E. A 64-year-old woman with primary hepatocellular carcinoma admitted to the hospital with acute renal failure.
+
++
The answer is D. (Chap. 368) Currently in the United States, all donor livers are distributed through a nationwide organ-sharing network (United Network for Organ Sharing [UNOS]) designed to allocate available organs based on regional considerations and recipient acuity. Recipients who have the highest disease severity generally have the highest priority, but allocation strategies that balance highest urgency against best outcomes continue to evolve to distribute cadaver organs most effectively. Allocation is based on the MELD score, which is based on a mathematical model that includes bilirubin, creatinine, and international normalized ratio. Neither waiting time (except as a tie breaker between two potential recipients with the same MELD scores) nor posttransplantation outcome is taken into account. Use of the MELD score has been shown to reduce waiting list mortality, to reduce waiting time prior to transplantation, and to be the best predictor of pretransplantation mortality. The highest priority (status 1) for liver transplantation continues to be reserved for patients with fulminant hepatic failure or primary graft nonfunction.
++
++
++
A 44-year-old woman is evaluated for complaints of abdominal pain. She describes the pain as a postprandial burning pain. It is worse with spicy or fatty foods and is relieved with antacids. She is diagnosed with a gastric ulcer and is treated appropriately for H pylori. During the course of her evaluation for her abdominal pain, the patient had a right upper quadrant ultrasound that demonstrated the presence of gallstones. Following treatment of H pylori, her symptoms have resolved. She is requesting your opinion regarding whether treatment is required for the finding of gallstone disease. Upon review of the ultrasound report, there were numerous stones in the gallbladder, including in the neck of the gallbladder. The largest stone measures 2.8 cm. What is your advice to the patient regarding the risk of complications and the need for definitive treatment?
++
++
++
A. Given the size and number of stones, prophylactic cholecystectomy is recommended.
++
++
B. No treatment is necessary unless the patient develops symptoms of biliary colic frequent and severe enough to interfere with the patient’s life.
++
++
C. The only reason to proceed with cholecystectomy is development of gallstone pancreatitis or cholangitis.
++
++
D. The risk of developing acute cholecystitis is about 5%–10% per year.
++
++
E. Ursodeoxycholic acid should be given at a dose of 10–15 mg/kg daily for a minimum of 6 months to dissolve the stones.
+
++
The answer is B. (Chap. 369) In the National Health and Nutrition Examination Survey, the prevalence of gallstone disease in the United States was 7.9% in men and 16.6% in women. Although the disease is quite prevalent, not all patients with gallstone disease require cholecystectomy. It is estimated that 1%–2% of patients with asymptomatic gallstone disease will develop complications that will require surgery yearly. Therefore, it is important to know which patients with asymptomatic gallstones require referral for surgery. The first factor to consider is whether the patient has symptoms that are caused by gallstones and are frequent enough and severe enough to necessitate surgery. Commonly called biliary colic, the classic symptoms of gallstone disease are right upper quadrant pain and fullness that begins suddenly and can last as long as 5 hours. Nausea and vomiting can accompany the episode. Vague symptoms of epigastric fullness, dyspepsia, and bloating following meals should not be considered biliary colic. A second factor that would be considered in recommending a patient for cholecystectomy is whether the patient has a prior history of complications of gallstone disease such as pancreatitis or acute cholecystitis. A final factor that would lead to the recommendation for cholecystectomy is the presence of anatomical factors that would increase the likelihood of complications such as a porcelain gallbladder or congenital abnormalities of the biliary tract. Individuals with very large stones (>3 cm) would also need to be considered carefully for cholecystectomy. Ursodeoxycholic acid can be used in some instances to dissolve gallstones. It acts to decrease the cholesterol saturation of bile and also allows dispersion of cholesterol from stones by producing a lamellar crystalline phase. However, it is only effective in individuals with radiolucent stones measuring less than 10 mm.
++
++
++
A 62-year-old man has been hospitalized in intensive care for the past 3 weeks following an automobile accident resulting in multiple long bone fractures and acute respiratory distress syndrome. He has been slowly improving but remains on mechanical ventilation. He is now febrile and hypotensive requiring vasopressors. He is being treated empirically with cefepime and vancomycin. Multiple blood cultures are negative. He has no new infiltrates or increasing secretions on chest radiograph. His laboratory studies demonstrated a rise in his liver function tests, bilirubin, and alkaline phosphatase. Amylase and lipase are normal. A right upper quadrant ultrasound shows sludge in the gallbladder but no stones. The bile duct is not dilated. What is the next best step in the evaluation and treatment of this patient?
++
++
++
++
++
B. Initiate treatment with clindamycin
++
++
C. Initiate treatment with metronidazole
++
++
D. Perform hepatobiliary scintigraphy
++
++
E. Refer for exploratory laparotomy
+
++
The answer is D. (Chap. 369) A practitioner needs to have a high index of suspicion for acalculous cholecystitis in critically ill patients who develop decompensation during the course of treatment for the underlying disease and have no other apparent source of infection. Some predisposing conditions for the development of acalculous cholecystitis include serious trauma or burns, postpartum following prolonged labor, prolonged parenteral hyperalimentation, and during the postoperative period following orthopedic and other major surgical procedures. The clinical manifestations of acalculous cholecystitis are identical to calculous disease, but the disease is more difficult to diagnose. Ultrasonography and CT scanning typically only show biliary sludge, but they may demonstrate large and tense gallbladders. Hepatobiliary scintigraphy often shows delayed or absent gallbladder emptying. Successful management relies on accurate and early diagnosis. In critically ill patients, a percutaneous cholecystostomy may be the safest immediate procedure to decompress an infected gallbladder. Once the patient is stabilized, early elective cholecystectomy should be considered. Metronidazole to provide anaerobic coverage should be added, but this would not elucidate or adequately treat the underlying condition.
++
++
++
All the following are associated with an increased risk for cholesterol stone cholelithiasis EXCEPT:
++
++
++
A. Gallbladder sludge on ultrasound
++
++
++
++
++
++
++
++
+
++
The answer is B. (Chap. 369) Gallstones are very common, particularly in Western countries, with cholesterol stones being responsible for >90% of cases of cholelithiasis and pigment stones account for the remaining <10%. Cholesterol is essentially water-insoluble. Stone formation occurs in the setting of factors that upset cholesterol balance. Obesity, cholesterol-rich diets, high-calorie diets, and certain medications affect biliary secretion of cholesterol. Intrinsic genetic mutations in certain populations may affect the processing and secretion of cholesterol in the liver. Pregnancy results in both an increase in cholesterol saturation during the third trimester and changes in gallbladder contractility. Although rapid weight loss and low-calorie diets are associated with gallstones, there is no evidence that a high-protein diet confers an added risk of cholelithiasis.
++
++
++
All of the following conditions are associated with an increased risk of pigment stone cholelithiasis EXCEPT:
++
++
++
++
++
B. Chronic biliary tract infection
++
++
C. Chronic hemolytic anemia
++
++
++
++
E. Primary biliary cirrhosis
+
++
The answer is E. (Chap. 369) Pigment stone cholelithiasis may be black or brown stones. Black pigment stones are composed of either pure calcium bilirubinate or polymer-like complexes with calcium and mucin glycoproteins. They are more common in patients who have chronic hemolytic states (with increased conjugated bilirubin in bile), liver cirrhosis, Gilbert syndrome, or cystic fibrosis. Gallbladder stones in patients with ileal diseases, ileal resection, or ileal bypass generally are also black pigment stones. Enterohepatic recycling of bilirubin in ileal disease states contributes to their pathogenesis. Brown pigment stones are composed of calcium salts of unconjugated bilirubin with varying amounts of cholesterol and protein. They are caused by the presence of increased amounts of unconjugated, insoluble bilirubin in bile that precipitates to form stones. Sometimes, the enzyme is also produced when bile is chronically infected by bacteria, and such stones are brown. Pigment stone formation is frequent in Asia and is often associated with infections in the gallbladder and biliary tree, including parasitic infections. Primary biliary cirrhosis is associated with cholesterol stones because of decreased bile acid secretion.
++
++
++
A 41-year-old woman presents to your clinic with a week of jaundice. She notes pruritus, icterus, and dark urine. She denies fever, abdominal pain, or weight loss. The examination is unremarkable except for yellow discoloration of the skin. Total bilirubin is 6.0 mg/dL, and direct bilirubin is 5.1 mg/dL. AST is 84 IU/L, and ALT is 92 IU/L. Alkaline phosphatase is 662 IU/L. CT scan of the abdomen is unremarkable. Right upper quadrant ultrasound shows a normal gallbladder but does not visualize the common bile duct. What is the most appropriate next management step?
++
++
++
A. Antibiotics and observation
++
++
B. Endoscopic retrograde cholangiopancreatography
++
++
++
++
D. Hepatobiliary iminodiacetic acid (HIDA) scan
++
++
E. Serologies for antimitochondrial antibodies
+
++
The answer is B. (Chaps. 58 and 369) The clinical presentation is consistent with a cholestatic picture. Painless jaundice always requires an extensive workup, as many of the underlying pathologies are ominous and early detection and intervention often offers the only hope for a good outcome. The gallbladder showed no evidence of stones, and the patient shows no evidence of clinical cholecystitis, and so a hepatobiliary iminodiacetic acid (HIDA) scan is not indicated. Similarly, antibiotics are not necessary at this point. The cholestatic picture without significant elevation of the transaminases on the liver function tests makes acute hepatitis unlikely. Antimitochondrial antibodies are elevated in cases of PBC, which may present in a similar fashion. However, PBC is far more common in women than in men, and the average age of onset is the fifth or sixth decade. The lack of an obvious lesion on CT scan does not rule out a source of the cholestasis in the biliary tree. Malignant causes, such as cholangiocarcinoma and tumor of the ampulla of Vater, and nonmalignant causes, such as sclerosing cholangitis and Caroli disease, may be detected only by direct visualization with endoscopic retrograde cholangiopancreatography (ERCP). ERCP is useful both diagnostically and therapeutically as stenting procedures may be done to alleviate the obstruction.
++
++
++
A 32-year-old woman is being evaluated for her second episode of acute-onset right upper quadrant pain and jaundice. She’s noticed worsening jaundice over the last 2 weeks and now complains of diffused itchiness. She has a 10-year history of ulcerative colitis treated with sulfasalazine. She is found to have a markedly elevated serum bilirubin and alkaline phosphatase. Her aminotransferases and prothrombin time are normal. Ultrasound shows no gallstones. Magnetic resonance cholangiopancreatography reveals a beaded appearance of her intrahepatic and extrahepatic bile ducts due to multiple discrete strictures. All of the following statements regarding her diagnosis are true EXCEPT:
++
++
++
A. Cyclosporine A is the most effective medical therapy.
++
++
B. Median survival with medical therapy is approximately 10 years.
++
++
C. She may be a candidate for liver transplantation.
++
++
D. Surgical therapy is rarely indicated.
++
++
E. The biliary disease is associated with her ulcerative colitis.
+
++
The answer is A. (Chap. 369) Primary or idiopathic sclerosing cholangitis (PSC) is characterized by a progressive, inflammatory, sclerosing, and obliterative process affecting the extrahepatic and/or the intrahepatic bile ducts. The disorder occurs in up to 75% of patients with inflammatory bowel disease, especially ulcerative colitis. It may also be associated with autoimmune pancreatitis; multifocal fibrosclerosis syndromes such as retroperitoneal, mediastinal, and/or periureteral fibrosis; Riedel struma; or pseudotumor of the orbit. Immunoglobulin G4 (IgG4)–associated cholangitis is a recently described biliary disease of unknown etiology that presents with biochemical and cholangiographic features indistinguishable from PSC, is often associated with autoimmune pancreatitis and other fibrosing conditions, and is characterized by elevated serum IgG4 and infiltration of IgG4-positive plasma cells in bile ducts and liver tissue. In contrast to PSC, it is not associated with inflammatory bowel disease and should be suspected if associated with increased serum IgG4 and unexplained pancreatic disease. Patients with PSC often present with signs and symptoms of chronic or intermittent biliary obstruction: right upper quadrant abdominal pain, pruritus, jaundice, or acute cholangitis. Late in the course, complete biliary obstruction, secondary biliary cirrhosis, hepatic failure, or portal hypertension with bleeding varices may occur. The diagnosis is usually established by finding multifocal, diffusely distributed strictures with intervening segments of normal or dilated ducts, producing a beaded appearance on cholangiography. Patients are at higher risk of cholangiocarcinoma. Therapy with cholestyramine may help control symptoms of pruritus, and antibiotics are useful when cholangitis complicates the clinical picture. Glucocorticoids, methotrexate, and cyclosporine have not been shown to be efficacious in PSC. In cases where high-grade biliary obstruction (dominant strictures) has occurred, balloon dilatation or stenting may be appropriate. Only rarely is surgical intervention indicated. The prognosis is unfavorable, with a median survival of 9–12 years following the diagnosis, regardless of therapy. Four variables (age, serum bilirubin level, histologic stage, and splenomegaly) predict survival in patients with PSC and serve as the basis for a risk score. PSC is a common indication for liver transplantation.
++
++
++
Which of the following is the most important ion secreted by the pancreas?
++
++
++
++
++
++
++
++
++
++
++
+
++
The answer is A. (Chap. 371) Bicarbonate is the ion of primary physiologic importance within pancreatic secretion. The ductal cells secrete bicarbonate predominantly derived from plasma (93%) more than from intracellular metabolism (7%). Bicarbonate enters the duct lumen through the sodium bicarbonate cotransporter with depolarization caused by chloride efflux through the cystic fibrosis transmembrane conductance regulator (CFTR). Secretin and vasoactive intestinal peptide bind at the basolateral surface and cause an increase in secondary messenger intracellular cyclic adenosine monophosphate, and act on the apical surface of the ductal cells, opening the CFTR and promoting secretion. CCK, acting as a neuromodulator, markedly potentiates the stimulatory effects of secretin. Acetylcholine also plays an important role in ductal cell secretion. Intraluminal bicarbonate secreted from the ductal cells helps neutralize gastric acid and creates the appropriate pH for the activity of pancreatic enzymes and bile salts on ingested food.
++
++
++
A 45-year-old woman with known history of cholelithiasis is admitted to the hospital with severe mid-epigastric pain, fever to 38.5°C, tachycardia to 110 bpm, and a blood pressure of 100/50 mmHg. Her examination shows a diffusely tender abdomen with guarding. Radiographs show an abdominal ileus with no free air. Laboratories are notable for a hemoglobin of 15 g/dL and elevations or amylase and lipase. Which of the following statements regarding this patient’s likely diagnosis is true?
++
++
++
A. Elevated lipase is more specific than elevated amylase for the diagnosis of acute pancreatitis.
++
++
B. Hypercalcemia occurs in >75% of cases of acute pancreatitis.
++
++
C. Magnitude of lipase elevation above normal is correlated with the severity of acute pancreatitis.
++
++
D. Serum amylase levels will remain elevated for up to 30 days after the resolution of acute pancreatitis.
++
++
E. The combination of elevated serum amylase and metabolic acidosis (pH <7.32) has a >90% positive predictive value for acute pancreatitis.
+
++
The answer is A. (Chap. 371) In combination with a consistent clinical story and radiographic findings, serum amylase and lipase values threefold or more above normal virtually clinch the diagnosis of acute pancreatitis. Gut perforation, ischemia, and infarction should be excluded. Serum lipase is the preferred test and has a higher specificity than serum amylase. There is no correlation between the severity of pancreatitis and the degree of serum lipase and amylase elevations. After 3–7 days, even with continuing evidence of pancreatitis, total serum amylase values tend to return toward normal. However, pancreatic isoamylase and lipase levels may remain elevated for 7–14 days. Elevation of serum amylase is not specific for acute pancreatitis; notably, patients with metabolic academia (e.g., diabetic ketoacidosis) may have spurious elevation of serum amylase without pancreatitis. Hypocalcemia occurs in approximately 25% of cases of acute pancreatitis, whereas hypercalcemia is not a feature.
++
++
++
A 27-year-old woman is admitted to the hospital with acute-onset severe right upper quadrant pain that radiates to the back. The pain is constant and not relieved with eating or bowel movements. Her labs show marked elevation in amylase and lipase, and acute pancreatitis is diagnosed. Which of the following is the best first test to demonstrate the etiology of her pancreatitis?
++
++
++
A. Right upper quadrant ultrasound
++
++
++
++
C. Serum triglyceride level
++
++
++
++
+
++
The answer is A. (Chap. 371) The most common cause of acute pancreatitis in the United States is gallstones causing common bile duct obstruction. Although bile duct obstruction may be demonstrated on technetium HIDA scan, right upper quadrant ultrasound is preferred for ease, demonstration of gallstones in the gallbladder, and demonstration of obstructed bile duct. Alcohol is the second most common cause, followed by complications of ERCP. Hypertriglyceridemia accounts for 1%–4% of cases with triglyceride levels usually >1000 mg/dL. Other potential common causes include trauma, surgery, drugs such as valproic acid, anti-HIV medications, estrogens, and sphincter of Oddi dysfunction. Additionally, a number of rare causes have been described. The most judicious first step in evaluation is to test for gallstones and pursue more rare causes after the most common cause has been ruled out.
++
++
++
A 58-year-old man with severe alcoholism is admitted to the hospital with acute pancreatitis. His symptoms have been present for 3 days, and he has persisted to drink heavily. He now has persistent vomiting and feels dizzy upon standing. On examination, he has severe epigastric and right upper quadrant tenderness and decreased bowel sounds, and he appears uncomfortable. A faint blue discoloration is present around the umbilicus. What is the significance of this finding?
++
++
++
A. A CT of the abdomen is likely to show severe necrotizing pancreatitis.
++
++
B. Abdominal plain film is likely to show pancreatic calcification.
++
++
C. Concomitant appendicitis should be ruled out.
++
++
D. He likely has a pancreatico-aortic fistula.
++
++
E. Pancreatic pseudocyst is likely present.
+
++
The answer is A. (Chap. 371) Physical examination in acute pancreatitis commonly shows an uncomfortable patient, often with low-grade fever, tachycardia, and hypotension. Abdominal tenderness and muscle rigidity are often present to varying degrees. Cullen sign is a faint blue discoloration around the umbilicus that may occur as the result of hemoperitoneum. Turner sign is blue-red-purple or green-brown discoloration of the flanks from tissue catabolism of hemoglobin. Both of these signs indicate the presence of severe necrotizing pancreatitis.
++
++
++
A 36-year-old man is admitted to the hospital with acute pancreatitis. In order to determine the severity of disease and risk of mortality, the Bedside Index of Severity in Acute Pancreatitis (BISAP) is calculated. All of the following variables are used to calculate this score EXCEPT:
++
++
++
++
++
B. Blood urea nitrogen >35
++
++
C. Impaired mental status
++
++
++
++
E. Serum lipase >3× normal
+
++
The answer is E. (Chap. 313) The BISAP (Bedside Index of Severity in Acute Pancreatitis) score has recently replaced Ranson’s criteria and Acute Physiology and Chronic Health Evaluation II (APACHE II) severity scores as the recommended modality to assess severity of pancreatitis due to the cumbersome nature of the prior scores and the requirement of prior scores to collect a large amount of clinical and laboratory data over time. Severity of acute pancreatitis should be determined in the emergency department to assist in patient triage to a regular hospital ward or step-down unit or direct admission to an intensive care unit. BISAP incorporates five clinical and laboratory parameters obtained within the first 24 hours of hospitalization—BUN >25 mg/dL, impaired mental status (Glasgow coma score <15), systemic inflammatory response syndrome, age >60 years, and pleural effusion on radiography—that can be useful in assessing severity (Table VIII-92). The presence of three or more of these factors was associated with substantially increased risk for in-hospital mortality among patients with acute pancreatitis. In addition, an elevated hematocrit >44% and admission BUN >22 mg/dL are also associated with more severe acute pancreatitis. Incorporating these indices with the overall patient response to initial fluid resuscitation in the emergency ward can be useful at triaging patients to the appropriate hospital acute care setting. Elevation of serum lipase is important for establishing the diagnosis of acute pancreatitis, but the degree of elevation is not correlated with severity of disease.
++
++
++
++
A 54-year-old man is admitted to the ICU with severe pancreatitis. His BMI is ≥30 kg/m2, and he has a prior history of diabetes mellitus. A CT of the abdomen is obtained and shows severe necrotizing pancreatitis. He is presently afebrile. Which of the following medications has been shown to be effective in the treatment of acute necrotizing pancreatitis?
++
++
++
++
++
++
++
++
++
++